8/2

Ace your homework & exams now with Quizwiz!

A 48-year-old man comes to the office due to right inguinal discomfort. The patient first noticed "bumps" in his groin when he was dressing in the morning. The right inguinal lymph nodes are enlarged and tender, as are several nodes in the right popliteal area. The distribution of lymphadenopathy in this patient would most likely be seen in which of the following conditions?

*Purulent laceration of the right lateral foot* The lymphatic system of the extremities is divided into the superficial lymphatic vessels, which follow the venous system, and the deep lymphatic vessels, which follow the arterial system. The superficial vessels receive lymph from the skin and subcutaneous tissues, whereas the deep vessels drain both the deep muscles and the superficial vessels. In the lower extremities, the superficial lymphatic system is divided into medial and lateral tracks. The medial track runs along the long saphenous vein up to the superficial inguinal lymph nodes, bypassing the popliteal nodes. Consequently, lesions on the medial foot cause inguinal lymphadenopathy (Choice C). In contrast, lateral lesions, which drain via the lateral track and communicate with the popliteal and inguinal nodes, are more likely to cause lymphadenopathy in both the popliteal and inguinal areas.

Opthalmic veins

- superior - communicates with facial vein - inferior - communicates with the pterygoid plexus - both drain through the superior orbital fissure and into the cavernous sinus

What is pathognomonic for asbestosis

Asbestosis can present with dyspnea on exertion, but chest x-ray is more likely to reveal an interstitial pattern of involvement most prominent in the lower zones. Pleural plaques may also be noted. Histology shows ferruginous bodies featuring fusiform rods with a translucent asbestos center and a golden-brown iron coating.

What is seen with berryliosis

Berylliosis may present with dyspnea and ill-defined nodular or irregular opacities on chest x-ray. Histology reveals noncaseating epithelioid granulomas without obvious associated particles.

Adherins and desmosomes are both made up of _________

Both adherens junctions and desmosomes are composed of cadherins and are involved in intercellular adhesion. The cytoplasmic anchor of adherens junctions is the actin filament, whereas the cytoplasmic anchor of desmosomes is the intermediate filament. Autoantibodies against desmoglein, a cadherin protein for desmosomes, are found in pemphigus vulgaris.

Patient started on Buproprion for MDD. However, she still feels "really down" at times and has a poor appetite. Further history suggests that the patient consumes very little food each day (a pattern that preceded her use of bupropion) and exercises excessively. Due to her ongoing mood problems, increasing the dose of her antidepressant medication is considered. Which of the following potential side effects of the increased dose would be of primary concern?

Bupropion is an antidepressant that works by inhibiting the reuptake of norepinephrine and dopamine. It is commonly used to treat major depression and is often preferred by patients because it does not cause weight gain or sexual side effects. Seizures are a potential side effect of bupropion therapy, especially when the medication is given at high doses. The risk is even greater in patients with pre-existing seizure disorders or eating disorders with potential electrolyte imbalances. Bupropion is therefore contraindicated in patients with seizure disorders or current or prior diagnosis of bulimia or anorexia nervosa. This patient's possible food restriction and excessive exercise suggest the possibility of an eating disorder, and the increased seizure risk with a higher dosage of bupropion would be of primary concern.

Which COX is invovled with vasodilation

COX 2 is expressed in vascular endothelial cells and vascular smooth muscle cells, where it plays a role in the local production of prostacyclin, which has anticoagulant and vasodilatory actions. COX 2 inhibitors have been associated with an increased risk of cardiovascular events.

What would be seen in the amniotic fluid of a pt with Down Syndrome

Chromosomal disjunction during meiosis involves separation of 23 pairs of homologous chromosomes in anaphase I, followed by separation of 23 pairs of sister chromatids during anaphase II. Advanced maternal age is a risk factor for Down syndrome (nondisjunction of chromosome 21), which presents with low AFP and normal AChE levels in the amniotic fluid.

Diminished proprioception and motor function can occur with....

Diminished proprioception and motor function can occur with simultaneous injury to the dorsal columns and CSTs. A classic cause is subacute combined degeneration of the dorsal and lateral SC white matter, which is seen in vitamin B12 deficiency.

Failure of the processus vaginalis to obliterate results in

Failure of the processus vaginalis to obliterate can lead to congenital hydrocele or indirect inguinal hernia.

What are fenestrae and what are they implicated in

Fenestrae are gaps between endothelial cells that allow for paracellular transport. Swollen fenestrae in renal glomerular capillary endothelial cells are implicated in preeclampsia (presents with new-onset hypertension and proteinuria or end-organ dysfunction at >20 weeks gestation).

What is Gilbert's syndrome?

Glucuronyl transferase (uridine 5'-diphospho-glucuronosyltransferase) is necessary for hepatic bilirubin conjugation. Gilbert syndrome, a condition marked by jaundice (elevated unconjugated bilirubin levels) during times of stress, results from mutations in the gene encoding glucuronyl transferase. mild unconjugated hyperbilirubinemia thought to be provoked by one of the classic triggers (fasting, stress, illness)

What are hemidesmosomes and what are they implicated in

Hemidesmosomes link cells to the basement membrane via integrins, the transmembrane anchor proteins. Autoantibodies to these proteins cause bullous pemphigoid and pemphigoid gestationis.

Failure of neural crest cell migration is seen in what condition?

Hirschsprung disease involves failure of neural crest cells to migrate into the sigmoid colon/rectum. It leads to absence of the myenteric and submucosal nervous plexuses, causing tonic constriction of the affected intestinal segments with consequent bilious emesis, abdominal distension, and failure to pass meconium.

what is Vernetts syndrome aka jugular foramen syndrome

Lesions of the jugular foramen can result in jugular foramen (Vernet) syndrome, which is characterized by the dysfunction of CNs IX, X, and XI. Findings include dysphagia, hoarseness, loss of gag reflex on the ipsilateral side, and deviation of the uvula toward the normal side

A 36-year-old immigrant from Peru comes to the office due to difficulty swallowing liquids. He also has difficulty belching. Eating slowly and extending the neck partially relieves his symptoms. Barium swallow shows a dilated esophagus, and manometry confirms absent peristalsis in the smooth muscle portion of the esophagus. If this patient's symptoms are caused by an infection, which of the following organisms is the most likely cause?

Trypanosoma Cruzi This patient who presents with dysphagia and a dilated esophagus has achalasia, which is characterized by the absence of distal esophageal peristalsis and incomplete relaxation of a hypertensive lower esophageal sphincter. It is most often a primary disorder; however, in a patient from Central or South America, secondary achalasia due to Chagas disease should be suspected. Chagas disease is caused by chronic infection with Trypanosoma cruzi, a slender C- or U-shaped flagellated parasite with darkly staining nucleus and kinetoplast. Parasitosis-related inflammation and immune-mediated cross-reactivity between the parasite and the enteric ganglia lead to destruction of the submucosal (Meissner) and myenteric (Auerbach) plexus. Denervation results in uncoordinated smooth muscle activity, increased esophageal tone, and incomplete lower esophageal relaxation. Mechanical dilation (due to obstructed liquids and solids) proximal to the functional obstruction manifests as megaesophagus. Patients classically experience dysphagia progressing from solids to liquids, odynophagia (due to food impaction), difficulty belching, regurgitation, and malnutrition. Other manifestations of Chagas disease can include nonischemic cardiomyopathy and megacolon. In addition, patients are at higher risk for esophageal cancer.

A 55-year-old woman comes for evaluation of persistent morning stiffness. She was diagnosed with rheumatoid arthritis 4 months ago and was prescribed methotrexate. The patient currently takes the maximum tolerated dose, along with folic acid and as-needed naproxen. Treatment with etanercept is considered. Which of the following tests should be performed before beginning treatment with this agent?

Tumor necrosis factor-alpha (TNF-α) inhibitors are large-molecule anti-inflammatory agents ("biologics") commonly used to treat moderate to severe rheumatoid arthritis, particularly in patients who have failed methotrexate therapy. Etanercept is a fusion protein with domains derived from the Fc portion of IgG1 and TNF receptor 2. It functions as a decoy receptor for TNF-α. Other TNF-α inhibitors are anti-TNF monoclonal antibodies (eg, infliximab, adalimumab). Inhibition of TNF-α leads to impaired cell-mediated immunity. In particular, TNF-α is necessary for effective sequestration of mycobacteria within granulomas. As a result, TNF-α inhibitors promote reactivation of latent tuberculosis and can increase the risk of disseminated disease. All patients being considered for TNF-α inhibitor therapy should have a baseline tuberculin skin test or interferon-gamma release assay to screen for latent tuberculosis. TNF-α inhibitors also increase susceptibility to other infectious agents, including fungi and atypical mycobacteria, and should not be used in any patient with an underlying infection.

Coal worker's pneumoconiosis presents with

can present with exertional dyspnea and nodular interstitial opacities on chest x-ray. Histology of nodal and perilymphatic lung tissue shows accumulations of black, carbon-laden macrophages (coal macules).

Pyruvate kinase deficiency

dec. ATP -> dehydrated RBC -> inc. 2,3-BPG shifts O2 curve to the right (make it easier to drop off O2) Pyruvate kinase deficiency is typically inherited in an autosomal recessive pattern and leads to hemolytic anemia. Pyruvate kinase deficiency can present with pallor, scleral icterus, and splenomegaly

What is hungry bone syndrome?

pt with severe hypercalcemia and hyperparathyroidism underwent parathyroid removal; then sudden withdrawn of PTH causing an influx of ca from circulation to bone; developing severe hypocalcemia; devleoped 2-3 days after surgery Hungry bone syndrome causes hypophosphatemia and hypocalcemia due to the rapid formation of bone after parathyroidectomy in a patient with chronic hyperparathyroidism. However, significant hypocalcemia typically results in tetany and this disorder usually occurs in the early postoperative period (2-4 days).

Patient with narcolepsy. What is best tx?

Narcolepsy is a rare neurologic disorder characterized by episodes of irresistible sleep during the day and usually one or more REM sleep-related phenomena such as cataplexy (ie, sudden weakness triggered by strong emotions), hypnagogic/hypnopompic hallucinations (ie, hallucinations while falling asleep or awakening, respectively), and sleep paralysis (ie, physiologic sleep paralysis that persists on awakening/falling asleep). The etiology of narcolepsy is believed to be due to low levels of the stimulatory neurotransmitter orexin (ie, hypocretin), which is involved in maintaining wakefulness and suppressing REM sleep-related phenomena. Treatment involves the use of agents that promote wakefulness (psychostimulants). Modafinil, a nonamphetamine stimulant, has become the first-line agent because it is effective and well tolerated and its misuse is rare. The mechanism of action is not well described, but modafinil may enhance dopaminergic signaling. Amphetamines are second-line agents due to their sympathomimetic side effects (eg, hypertension, arrhythmia, psychosis) and risk for misuse and dependency.

A 3-year-old boy is brought to the emergency department for refusal to move his right arm. This afternoon, he tripped while holding hands with his sister, who pulled up on his arm to prevent him from falling. The patient immediately began to cry and has since refused to move the arm. On physical examination, he holds the right arm against his abdomen with the elbow flexed and forearm pronated. The patient cries with any attempted manipulation of the right elbow. There is no visible swelling or deformity. Which of the following structures is most likely affected in this patient's condition?

*Annular ligament* This patient's mechanism of injury (eg, sudden pulling up on the arm) and presentation (eg, holding the arm flexed and pronated, pain with movement) are consistent with radial head subluxation (ie, nursemaid's elbow). The radial head is anchored to the ulna by the annular ligament, which encircles the proximal radius. In children age <5, this tissue is relatively thin compared to older children and adults. Sudden axial traction (eg, pulling up on the arm by the hand) or twisting can cause the annular ligament to slip over the radial head and become entrapped in the radiohumeral joint. Patients classically hold the affected arm close to the body with the elbow flexed and the forearm pronated. There is significant pain with movement or manipulation of the joint but typically no obvious swelling or deformity. Reduction can usually be accomplished either by hyperpronation of the forearm or supination of the forearm with flexion of the elbow.

What structure returns CSF to the venous circulation

*Arachnoid villi* Once formed, CSF flows from the lateral ventricles to the third ventricle via the interventricular foramina of Monro. It then circulates to the fourth ventricle through the cerebral aqueduct (of Sylvius). CSF then exits the fourth ventricle and enters the subarachnoid space via one of 3 foramina: the 2 lateral foramina of Luschka and the single midline foramen of Magendie. CSF is returned to the venous circulation via the arachnoid villi (granulations).

Allopurinol is prescribed. Initiation of this drug would most likely increase the activity of which of the following medications?

*Azathioprine* In the purine degradation pathway, purine bases (ie, hypoxanthine, guanine) can follow 1 of 2 routes: Approximately 10% are converted by xanthine oxidase to uric acid, which is released into the bloodstream and excreted in the urine. The other 90% are recycled into purine nucleotides by hypoxanthine-guanine phosphoribosyltransferase (HGPRT) to be reused in the formation of DNA and RNA (ie, purine salvage). Gout occurs due to elevated serum uric acid levels, leading to the formation of inflammation-inducing uric acid crystals within joints (eg, metatarsals). Xanthine oxidase inhibitors (eg, allopurinol, febuxostat) treat gout by reducing uric acid levels and in the process shunt additional purine bases toward the salvage route. Azathioprine is an immunosuppression drug that is metabolized via the purine degradation pathway. Following ingestion, azathioprine is converted to 6-mercaptopurine (6-MP), which is a false purine base that, like hypoxanthine and guanine, can follow 1 of 2 routes. Some 6-MP is converted by xanthine oxidase (or thiopurine methyltransferase) into inactive metabolites that are excreted in the urine. The remainder is converted by HGPRT into active metabolites (6-thioguanines), which act as false nucleotides that incorporate into DNA and RNA and inhibit hematopoietic cell proliferation. Therefore, when a xanthine oxidase inhibitor is combined with azathioprine, 6-MP is shunted toward conversion into active metabolites, leading to increased immunosuppression and risk of cellular toxicity.

A 67-year-old man comes to the hospital due to an acute worsening of chronic back pain associated with new-onset bilateral leg weakness and numbness. He has a history of colorectal cancer with vertebral metastases for which he has received colectomy, chemotherapy, and radiation treatment. The patient takes oral morphine sulfate for his back pain. Examination shows focal vertebral tenderness, along with reduced muscular strength, decreased sensitivity to light touch, and spasticity in both legs. Imaging studies reveal a vertebral compression fracture with cord compression. Surgical decompression is planned. Which of the following drugs may precipitate opioid withdrawal symptoms if used in this patient?

*Buprenorphine* Opioids work by selectively binding to G protein-coupled opioid receptors, mimicking the actions of endogenous opioid peptides. However, long-term activation of mu-opioid receptors on nociception-transmitting neurons is associated with increased pain sensitivity (central sensitization) due to: 1. Increased turnover of inhibitory opioid receptors (receptor downregulation) and decoupling of receptors from their second messenger system (receptor decoupling) 2. Upregulation of excitatory N-methyl-D-aspartate (NMDA) receptors This manifests as an increasing dose requirement to provide the same level of pain relief (tolerance) and pain sensations triggered by benign stimuli (opioid-induced hyperalgesia). Opioid withdrawal can occur within 24-48 hours of acute cessation in opioid-tolerant patients and is characterized by nausea/vomiting, myalgias, dilated pupils, diaphoresis, and tachycardia. Buprenorphine is a partial opioid agonist that has low intrinsic activity (efficacy) for opioid mu-receptors. However, it binds with high affinity (potency) and can prevent binding of other opioid medications. Therefore, buprenorphine acts as an opioid receptor antagonist in the presence of full opioid agonists and can precipitate withdrawal in opioid-tolerant patients with chronic pain.

A 9-year old girl is brought to the emergency department by her parents due to severe headache, lethargy, and vomiting. Her symptoms began a few weeks ago with mild intermittent headaches and fatigue and have progressively worsened. A CT scan of the brain reveals a mass lesion, enlarged lateral and third ventricles, and a normal-sized fourth ventricle. Which of the following is the most likely site of obstruction in this patient?

*Cerebral aqueduct* Cerebrospinal fluid (CSF) is formed by 4 choroid plexuses found in each of the ventricles. The choroid plexus is composed of ependymal cell-covered outgrowths of pia mater capillaries that are fenestrated unlike other central nervous system (CNS) capillaries. The ependymal layer is a simple ciliated columnar epithelium with tight junctions between the cells. This selectively allows only certain materials from the blood to enter the CSF. Once formed, CSF flows from the lateral ventricles to the third ventricle via the interventricular foramina of Monro. It then circulates to the fourth ventricle through the cerebral aqueduct (of Sylvius). CSF then exits the fourth ventricle and enters the subarachnoid space via one of 3 foramina: the 2 lateral foramina of Luschka and the single midline foramen of Magendie. CSF is returned to the venous circulation via the arachnoid villi (granulations). This patient has findings consistent with noncommunicating (obstructive) hydrocephalus. Obstruction of CSF flow is most commonly caused by tumors, scarring (post-hemorrhage, infection), and CNS malformations. In this patient, obstruction at the level of the cerebral aqueduct causes increased pressures and ventricular dilation proximal to the site of blockage, which includes the lateral and third ventricles. The fourth ventricle is normal-sized as it is distal to the obstruction. In contrast, obstruction at the foramen of Magendie and Luschka would cause enlargement of all 4 ventriclesm while obstruction at the foramen of Monro would cause enlargement of only the affected lateral ventricle,

An 18-month-old boy is brought to the office due to language regression. He said several words at his 1-year appointment but now no longer speaks any words at all. His moods have also become more unpredictable over the past 4 months with frequent tantrums. The parents tried to bring him in sooner for evaluation, but they live in an impoverished part of the city and experienced financial difficulties with transportation to the office. On physical examination, the boy is quiet and maintains appropriate eye contact throughout the visit. Hemoglobin is 9 g/dL. Which of the following enzymes is most likely inhibited in this patient?

*D-Ala dehydratase inhibition* This child's language regression and anemia are most likely due to lead poisoning. Lead toxicity is most prevalent among impoverished children residing in deteriorating urban housing built before 1978. Young children are particularly susceptible to lead poisoning via inhalation and ingestion of lead-based paint dust or chips due to normal crawling and mouthing behaviors. The incomplete blood-brain-barrier in children is vulnerable to the neurotoxic effects of lead, which include long-standing behavioral problems and developmental delay or regression. Anemia in lead poisoning results from inhibition of ferrochelatase and δ-aminolevulinic acid (ALA) dehydratase in the heme biosynthesis pathway. Because protoporphyrin IX cannot combine with iron (Fe2+) to form heme due to ferrochelatase inhibition, it instead incorporates a zinc ion, leading to elevated zinc protoporphyrin levels. In addition, ALA levels are increased. Lead poisoning also commonly coexists with iron deficiency anemia, and severe lead poisoning can also induce hemolytic anemia.

Initial noncontrast head CT scan reveals a small temporal lobe hemorrhage. Over the next 24 hours, the patient becomes progressively obtunded and develops generalized tonic-clonic seizures. Blood pressure is 190/96 mm Hg, pulse is 38/min, and respirations are 6/min with brief episodes of apnea. A repeat noncontrast head CT scan reveals acute hemorrhage expansion with brain herniation. Which of the following findings is most likely to be observed on this patient's repeat neurologic examination?

*Dilated pupil* This patient developed a spontaneous intracerebral hemorrhage with subsequent clinical deterioration due to ongoing bleeding and hemorrhage expansion. Expanding space-occupying lesions (eg, hemorrhage, hematoma) increase intracranial pressure, which can manifest with headache, vomiting, impaired consciousness, and ultimately Cushing triad (eg, hypertension, bradycardia, bradypnea) when brain herniation is imminent. Herniation can occur at different portions of the brain depending on the location of the initial lesion. Lesions in the temporal lobe tend to cause transtentorial herniation of the uncus, which can compress the ipsilateral oculomotor nerve (CN III) as it exits the midbrain at the same level. Patients often develop ipsilateral oculomotor nerve palsy with a fixed dilated pupil due to preganglionic parasympathetic nerve fiber damage. If the intracranial pressure continues to increase, the herniation can progress to involve the uncus bilaterally and ultimately cause downward displacement of the brainstem (ie, central herniation). This results in continued clinical deterioration with irregular breathing (ie, Cheyne-Stokes respiration), abnormal posturing, and either small or mid-fixed pupils depending on the extent of the damage.

What peptide is upregulated in chronic kidney disease that directly inhibits 1-alpha hydroxylase from converting 25 dihydroxyvitamin D to 1-25, dihydroxyvitamin D?

*FGF23* The activity of 1-alpha-hydroxylase is upregulated by parathyroid hormone (PTH) and inhibited by fibroblast growth factor 23 (FGF23), a peptide secreted by osteocytes in response to high levels of 1,25-dihydroxyvitamin D. Chronic kidney disease (CKD) results in impaired conversion of 25-hydroxyvitamin D to 1,25-dihydroxyvitamin D due to the following factors: FGF23 levels increase early in CKD, causing direct inhibition of 1-alpha-hydroxylase Reduced glomerular filtration limits the delivery of 25-hydroxyvitamin D to proximal tubule cells Reduced functional renal mass limits production of 1-alpha-hydroxylase Inadequate production of 1,25-dihydroxyvitamin D causes reduced intestinal absorption of calcium. This, along with increased phosphate retention (due to impaired glomerular and tubular function), leads to a compensatory increase in parathyroid hormone secretion (secondary hyperparathyroidism). Chronically increased PTH levels have a deleterious effect on bone metabolism (renal osteodystrophy), and can cause weakness, bone and muscle pain, defective bone mineralization, and an increased risk of fractures.

A 46-year-old woman is brought by ambulance to the emergency department following a ground-level fall at home. She slipped while getting out of the shower and hit her left knee on the tile floor. The patient developed severe left knee pain and swelling and is unable to ambulate. Examination shows a contusion and soft tissue swelling at the knee with a large knee effusion. This patient has a comminuted patella fracture. Which of the following examination findings would most likely be seen in this patient?

*Inability to extend the knee against gravity* The patella is a large sesamoid bone that facilitates knee extension, protects the joint from direct injury, and improves nourishment of the distal femur articular cartilage. The quadriceps tendon inserts at the superior pole of the patella and wraps distally around the patella to become the patellar tendon, which inserts at the tibial tuberosity. Patella fractures are most commonly due to a direct blow to the anterior aspect of the knee (eg, landing on a flexed knee in a fall, striking the knee against the dashboard in a frontal motor vehicle collision). The patella can also be fractured indirectly due to excessive force transmitted through the quadriceps tendon (eg, landing on the feet after falling from a height). Patients develop acute swelling (often with an associated effusion), focal tenderness, inability to extend the knee against gravity, and a palpable gap in the extensor mechanism.

Scientists studying the kidney's response to hypoperfusion apply a clip to a pig's right renal artery that reduces blood flow to the kidney by about 70%. After 6 months, they perform a right nephrectomy and examine the glomeruli and tubules microscopically. Which of the following cell types would be most likely to undergo hyperplasia as a result of the clip placement?

*Modified smooth muscle cells of the afferent arteriole aka JG cells* A markedly narrowed renal artery (eg, due to an atherosclerotic plaque or external clip) cannot supply the kidney with enough blood to maintain a normal glomerular filtration rate. The reduced blood flow is sensed by the juxtaglomerular (JG) apparatus, which consists of the macula densa, extraglomerular mesangial cells (ie, Lacis cells), and JG cells. Macula densa cells are tall, narrow cells located in the distal tubule that monitor salt content and tubular flow rate. This information is transmitted to nearby JG cells that are located mainly in the wall of the afferent arteriole. JG cells are modified smooth muscle cells with renin-containing zymogen granules. Significant renal hypoperfusion leads to a compensatory increase in renin secretion by JG cells. This activates the renin-angiotensin-aldosterone system, leading to increased levels of angiotensin II and aldosterone. Long-term renal hypoperfusion, such as caused by renal artery stenosis, leads the JG cells of the affected kidney(s) to undergo hyperplasia as a result of chronic stimulation. When renal artery stenosis is restricted to one side and the contralateral kidney functions normally, chronic kidney disease does not occur as the normal kidney is still able to efficiently filter and excrete waste products (eg, creatinine, urea). However, hypertension can occur in both unilateral and bilateral disease, as secretion of renin by one kidney will lead to systemic vasoconstriction and retention of salt and water by both kidneys.

A 28-year-old woman is treated with high-dose prednisone for severe lupus nephritis. Several hours after therapy is initiated, she becomes very agitated and delusional. Blood pressure is 130/70 mm Hg and heart rate is 110/min. A basic metabolic profile, complete blood cell (CBC) count, and urinalysis are obtained. The CBC differential is expected to show an increase in which of the following as a result of this patient's therapy?

*Neutrophils* As a result of their immunosuppressive effects, corticosteroids such as prednisone have been used to treat many autoimmune and inflammatory conditions, including systemic lupus erythematous. However, corticosteroid use can lead to a number of adverse effects. High doses can sometimes cause corticosteroid-induced psychosis (confusion, hallucinations), as seen in this patient; hypoalbuminemia is a risk factor, and the neuropsychiatric symptoms typically resolve with discontinuation of therapy. Corticosteroid receptors also have widespread physiologic effects, including those on circulating leukocytes and vascular endothelial cells. Neutrophil counts increase following administration of the drug as a result of "demargination" of neutrophils previously attached to the vessel wall. Therefore, neutrophil recruitment to fight infection in tissues is decreased, potentially contributing to increased infection risk.

For the past several weeks, the patient has had fluctuating tingling and numbness involving the right shoulder, arm, and hand. He also has recently developed dull pain in the right little finger and hand. The symptoms worsen with overhead activities and when throwing a baseball. The patient has no medical problems other than being told that he has an extra rib. On examination, there is right-sided weakness of the intrinsic hand muscles. Which of the following muscles is most likely contributing to this patient's condition?

*Scalene Muscles* This patient's upper extremity neurologic symptoms in the setting of an extra rib are highly suggestive of thoracic outlet syndrome (TOS). The thoracic outlet refers to the region above the first rib and behind the clavicle that is bordered by the cervical vertebral bones and the sternum. Compression of the brachial plexus as it passes through the thoracic outlet can cause upper extremity numbness, tingling, and weakness (most often in an ulnar distribution due to compression of the lower trunk). Involvement of the subclavian vein and artery can also occur and lead to upper extremity swelling and exertional arm pain. TOS most commonly occurs due to compression of the brachial plexus within the scalene triangle, which is formed by the anterior and middle scalene muscles and the first rib. The anterior scalene muscle originates from the C3-C6 transverse processes and attaches to the scalene tubercle of the first rib. The middle scalene originates from the C2-C7 transverse processes and inserts into the posterior portion of the first rib. The brachial plexus trunks and subclavian artery pass between the anterior and middle scalenes; the subclavian vein runs anterior to the anterior scalene. The presence of an anomalous cervical rib or scalene muscular anomalies can predispose to TOS. Affected patients typically have a history of trauma or repetitive overhead arm movements (eg, swimming, stacking boxes).

A 53-year-old man comes to the office due to an 8-month history of progressive shortness of breath that peaks during strenuous activity. Several of his coworkers have experienced similar symptoms. His medical history includes hypertension and type 2 diabetes mellitus. Pulmonary examination reveals diffuse fine crackles. Chest x-ray reveals nodular densities in both lungs that are most prominent in the apical regions. Calcification of the hilar lymph nodes is also seen. Bronchoscopy with transbronchial biopsy of a calcified node is performed, and polarized microscopy shows birefringent particles surrounded by dense collagen fibers. This patient most likely has a history of exposure to which of the following substances?

*Silica This middle-aged patient with dyspnea on exertion, nodular densities on x-ray, calcified hilar lymph nodes, and birefringent particles on biopsy has silicosis. Silicosis is a form of pneumoconiosis (ie, a type of interstitial lung disease caused by inhalation of mineral dust). Inhaled crystalline silica, typically due to industrial exposure (eg, mining, sand blasting), is toxic to alveolar macrophages and promotes the formation of intrapulmonary free radicals, leading to progressive lung injury and interstitial collagen deposition. Histologically, silicosis is characterized by birefringent silicate particles within dense, whorled collagenous nodules surrounded by dust-laden macrophages. Silicosis is often initially asymptomatic but can present with dyspnea on exertion and productive cough, typically 10-20 years after initial exposure. Radiography varies based on disease progression; simple silicosis typically demonstrates numerous small, rounded nodules predominant in the upper lobes that may rarely coalesce to form mass-like upper lobe fibrosis (progressive massive fibrosis). Calcification of the rim of hilar nodes (eggshell calcification) may also be seen.

A 10-year-old boy is brought to the emergency department after experiencing high fevers and chills for the last few days. He also complains of dull pain just above his left knee. He has no history of recent trauma other than minor scrapes to his knees and elbows while playing outside. Physical examination shows point tenderness 3 cm above the kneecap. There is no joint effusion. Radiographs show soft-tissue swelling and a periosteal reaction over the lower end of the femur. Which of the following organisms is most likely responsible for this patient's symptoms?

*Staph Auerus Osteomyelitis is an infection of bone and bone marrow that occurs by 1 of 3 mechanisms: 1. Hematogenous seeding due to an episode of bacteremia 2. Spread from a contiguous focus of infection, as occurs in an infected diabetic foot wound 3. Direct inoculation of bone, such as with a compound fracture Hematogenous osteomyelitis occurs predominantly in children (particularly boys) and most frequently affects the long bones. The tibia, fibula, and femur are most often involved. Adults who develop the condition are more likely to have vertebral involvement and frequently have a predisposition to bacteremia due to risk factors such as IV drug abuse or indwelling vascular catheters. The presenting symptoms of hematogenous osteomyelitis are vague, and a high index of suspicion is required to make the diagnosis. Initial symptoms such as malaise and fevers are non-specific. As the infection progresses, infants and younger children may refuse to move the affected extremity. Older children often complain of pain over a long bone. Bone pain develops as the abscess expands within the bone, leading to bone necrosis, periosteal disruption, and swelling of the surrounding soft tissue. Staphylococcus aureus is implicated in most cases of acute hematogenous osteomyelitis in otherwise healthy children.

Physical examination reveals localized tenderness just below the acromion. The physician asks the patient to abduct his arms 90 degrees to the side and flex them 30 degrees forward with his thumbs pointing to the floor. She then applies downward force to his arms. This maneuver elicits pain in the patient's right shoulder and reveals right-sided weakness as compared to the left side. A tendon of which of the following muscles is most likely injured in this patient?

*Supraspinatus The rotator cuff consists of the tendons of the supraspinatus, infraspinatus, teres minor, and subscapularis (SITS) and contributes to the stability and motion of the glenohumeral joint. During abduction of the humerus, the supraspinatus initiates movement through the first 10-15 degrees; subsequently, the deltoid provides the primary abductive force while the supraspinatus provides stability to the joint. Of all the rotator cuff structures, the supraspinatus tendon is most commonly affected in rotator cuff syndrome. This tendon is vulnerable to chronic repeated trauma from impingement between the head of the humerus and the acromion during abduction. Inflammation and fibrosis can worsen the problem by increasing friction between the head of the humerus and the acromion, as well as causing inflammation of the subacromial bursa. On examination, the action of the supraspinatus can be isolated with the "empty-can" supraspinatus test: abduction of the humerus in parallel to the axis of the scapula (30 degrees forward flexion) while in full internal rotation (thumbs pointed to the floor).

Physiologists are studying the biomolecular mechanisms underlying skeletal muscle contraction. They have been analyzing muscle fibers obtained from knockout mice to determine how different cellular substances influence muscular contraction. Striated muscle fibers obtained from a specific mouse embryo fail to contract in response to a substance normally released from the sarcoplasmic reticulum. A deficiency of which of the following would best explain this finding?

*Troponin* The contractile mechanism in skeletal muscle depends on proteins (myosin II, actin, tropomyosin, and troponin) as well as calcium ions. The thick filaments in skeletal muscle are comprised of myosin molecules, with the heads of the myosin molecules forming cross-links with actin during muscular contraction. Two actin chains comprise the thin filaments in skeletal muscle. Tropomyosin molecules sit in the groove between the two actin chains, covering the myosin binding sites on actin when the muscle is at rest. Troponin molecules are small globular proteins situated alongside the tropomyosin molecules. Troponin is composed of three subunits: troponin T, troponin I, and troponin C. Troponin T binds the other troponin components to tropomyosin, troponin I binds the troponin-tropomyosin complex to actin, and troponin C contains the binding sites for Ca2+. During excitation-contraction coupling, Ca2+ is released from the sarcoplasmic reticulum. When Ca2+ binds troponin C, tropomyosin shifts to expose the actin binding sites for myosin, allowing contraction to occur.

A 69-year-old man comes to the office due to progressive pain beneath his right scapula and in his right arm for the last several months. He has taken over-the-counter analgesics, which provide short-term relief, but the pain has become more severe and awakens him at night. The pain is associated with numbness in his right forearm extending up to the tips of the fourth and fifth fingers. The patient has also had persistent nonproductive cough and lately has been coughing up streaks of blood. He has a history of hypertension, gastroesophageal reflux disease, and osteoarthritis. He has smoked 2 packs of cigarettes daily for more than 40 years but recently cut down to 4 or 5 cigarettes a day. Which of the following is the most likely cause of this patient's symptoms?

*superior sulcus tumor* This patient with an extensive smoking history who now has hemoptysis and shoulder pain likely has lung cancer, which can be associated with symptoms due to local or distant spread. Pancoast syndrome is caused by a tumor at the lung apex. Such tumors often arise in the superior sulcus (groove formed by the subclavian vessels). The apical location allows for extensive local tumor spread. Invasion of surrounding structures can cause the following: Shoulder pain radiating toward the axilla and scapula (most common presenting symptom). It occurs due to involvement of the lower brachial plexus. Other associated symptoms include arm paresthesia, weakness, and muscle atrophy. Horner syndrome occurs due to involvement of the cervical sympathetic ganglia. Symptoms include ipsilateral ptosis, miosis, and anhydrosis. Upper extremity edema may be due to compression of subclavian vessels. Spinal cord compression and paraplegia can result from tumor extension into the intervertebral foramina

Patient with schizophrenia who has been on medication for years recently exhibits facial grimacing, lip smacking, and twisting movements of her hands and feet. On mental status examination, the patient makes poor eye contact and is generally distrustful but has no specific delusions or hallucinations. Which of the following is the most likely diagnosis?

*tardive dyskinesia* This patient has a diagnosis of schizophrenia and has likely been treated with antipsychotic medications for many years. Her abnormal movements are characteristic of tardive dyskinesia (TD), a movement disorder associated with long-term treatment with dopamine antagonist medications (eg, antipsychotics, metoclopramide). Possible mechanisms of TD include development of hypersensitive post-synaptic D2 receptors following prolonged D2 blockade and an imbalance between D1 and D2 receptor-mediated effects. TD typically consists of repetitive, rhythmic, involuntary movements of the tongue, lips, face, trunk, and extremities. Orofacial dyskinesias (eg, facial grimacing, tongue movements, lip smacking and puckering) are most common, but choreoathetoid movements of the trunk and limbs can also occur. Older age and exposure to first-generation antipsychotics increase the risk of developing TD. Treatment typically includes discontinuing the offending agent when feasible.

What are congenital melanocytic nevi

Congenital melanocytic nevi are caused by the clonal proliferation of benign melanocytes in utero. They can be raised and are hyperpigmented.

A 35-year-old man with a known history of peptic ulcer disease comes to the physician with sudden onset of pain, swelling, and redness at the base of his great toe. The patient was awakened in the middle of the night by the pain and was unable to go back to sleep. He has no history of trauma to the joint. Physical examination shows swelling, erythema, and exquisite tenderness involving the right first metatarsophalangeal joint. Fine-needle aspiration of the joint shows needle-shaped, negatively birefringent crystals. After making the diagnosis, the physician prescribes an appropriate medication. Shortly after starting the medication, the patient develops nausea, vomiting, and diarrhea. Which of the following is the most likely mechanism of action of the drug prescribed to this patient?

Acute onset of pain and swelling of the first metatarsophalangeal joint and joint aspiration showing needle-shaped, negatively birefringent crystals are diagnostic for acute gouty arthritis. Nonsteroidal anti-inflammatory drugs (NSAIDs) are the mainstay of treatment for most patients. However, colchicine is often used in patients with mild-to-moderate renal failure, peptic ulcer disease, or other contraindications to NSAIDs. Colchicine is an effective anti-inflammatory agent in acute gouty arthritis and acts by binding to the intracellular protein tubulin, preventing tubulin polymerization into microtubules. This leads to impaired leukocyte migration and phagocytosis, reducing the inflammation seen in gouty arthritis. Because colchicine also disrupts microtubule formation in gastrointestinal mucosal cells, many patients develop diarrhea and, less commonly, nausea, vomiting, and abdominal pain.

Agranulocytosis is a concering side effect associated w/ what psych drug

Agranulocytosis is a side effect related to treatment with the antipsychotic clozapine. The mood-stabilizing agents carbamazepine and valproate can also be associated with bone marrow abnormalities.

A 30-year-old man comes to the office due to depression. He describes a 4-week history of severely depressed mood, loss of motivation, loss of interest, and sleeping 12 hours a day. The patient has difficulty concentrating and worries that it is interfering with his work. He is pessimistic about the future but has no suicidal thoughts. The patient had a previous depressive episode in his 20s that responded rapidly to sertraline. He felt "really good and energetic" after a few days of treatment but discontinued the antidepressant a week later as he did not feel that he needed it. He drinks 1 or 2 beers on social occasions. Physical examination is normal. Which of the following would be most important to assess prior to initiating pharmacologic treatment for this patient?

All patients with a major depressive episode should be screened for a past history of manic episodes to differentiate major depressive disorder (unipolar depression) from bipolar disorder. This is particularly relevant in this patient as his atypically rapid response to the selective serotonin reuptake inhibitor sertraline is suggestive of treatment-emergent mania. If further history substantiates a diagnosis of bipolar disorder, antidepressant monotherapy should be avoided, as all antidepressants carry a risk of inducing mania. Bipolar patients in the depressed phase may be misdiagnosed with unipolar major depressive disorder when either the physician neglects to ask about past manic symptoms or patients do not report them due to limited insight, difficulty recalling, or a tendency to minimize past manic episodes. Physicians should question patients regarding a history of distinct periods of elevated mood and increased energy, decreased need for sleep, hyperactivity, racing thoughts, and uncharacteristic risk-taking behavior.

A 48-year-old man comes to the hospital after a day of high-grade fever, progressive headache, and double vision. The patient has been having purulent nasal drainage and frontal headache for the past several days. He has a history of type 2 diabetes mellitus. His temperature is 38.9 C (102 F), blood pressure is 110/70 mm Hg, and pulse is 94/min. Physical examination shows ptosis, mydriasis, mild proptosis, and loss of the corneal reflex of the right eye. Visual acuity is normal in both eyes, but the patient is unable to move the right eye in any direction. There is decreased sensation in the right upper face. Which of the following structures is most likely involved in this patient's presentation?

Cavernous sinus thrombosis is most commonly due to contiguous spread of an infection from the medial third of the face, sinuses (ethmoidal or sphenoidal), or teeth. The infection can communicate in a retrograde fashion into the cavernous sinus through the valveless facial venous system(via the superior and inferior ophthalmic veins). Common pathogens include Staphylococcus aureus (most common) and streptococci, although fungal organisms (eg, Mucor, Rhizopus) are responsible in rare cases. Patients typically experience headache, fever, and diplopia. Ocular muscle paralysis occurs due to injury of cranial nerves (CNs) III, IV, and VI. CN III palsy may also result in ptosis and mydriasis. Involvement of the ophthalmic and maxillary branches of CN V can cause loss of both upper facial sensation and the afferent limb of the corneal reflex. Proptosis (eye protrusion) and chemosis (conjunctival swelling) may also be evident due to impaired venous drainage through the ophthalmic veins.

A 58-year-old man undergoes operative repair of a descending thoracic aortic aneurysm. There are no intraoperative complications, but postoperatively the patient is found to have new-onset neurologic deficits. Imaging of the brain is normal, but MRI of the lower thoracic spinal cord reveals ischemic infarct of the anterior 2/3 of the spinal cord including spinthoalamic tract, anterior horn, and lateral corticospinal tract. Crude touch: Inc/Dec/Normal Pain sensation: Inc/Dec/Normal Proprioception: Inc/Dec/Normal Motor function: Inc/Dec/Normal

Crude touch: Dec Pain sensation: Dec Proprioception:Normal Motor function: Dec This patient's thoracic aortic aneurysm repair has been complicated by ischemic injury to the anterior two-thirds of the spinal cord (SC). This area is perfused by the anterior spinal artery, which receives blood flow from segmental arteries (eg, intercostal arteries) arising off the thoracic aorta. Disruption of circulation to this area classically results in anterior cord syndrome, characterized by the following bilateral tract lesions and clinical signs: 1. Spinothalamic tract (STT): loss of pain/temperature (lateral spinothalamic) and crude touch (anterior spinothalamic) sensation 1-2 levels below the lesion 2. Anterior horn (lower motor neuron): flaccid paralysis and muscle atrophy at the level of the lesion 3. Lateral corticospinal tract (CST) (upper motor neuron): paralysis below the level of the lesion. Flaccid paralysis from spinal shock (ie, temporary loss of all distal SC function) occurs initially, but upper motor neuron signs (eg, spasticity, hyperreflexia) subsequently develop over days to weeks As in this patient, the dorsal columns are typically spared (due to intact circulation from the posterior spinal arteries), preserving proprioceptive, vibratory, and light touch sensation below the level of the lesion.

The patient is started on celecoxib therapy. The presence of which of the following conditions in this patient's medical history would make this medication the preferred treatment?

Cyclooxygenase (COX; prostaglandin synthase) initiates the synthesis of prostanoids (eg, prostaglandins, thromboxane) from arachidonic acid. COX exists in 2 primary forms: 1. COX 1, which plays a physiologic role in a number of normally functioning tissues (eg, platelets, gastrointestinal tract) 2. COX 2, which is preferentially expressed at sites of inflammation. Nonsteroidal anti-inflammatory drugs (NSAIDs) work by inhibiting COX to block prostaglandin synthesis. Most NSAIDs inhibit both the COX 1 and COX 2 isoenzymes. However, inhibition of COX 1 may lead to severe adverse effects, including gastric ulceration (reduced mucosal protection) and increased bleeding (decreased platelet aggregation). Selective COX 2 inhibitors (eg, celecoxib) have potent anti-inflammatory effects with less risk of bleeding and gastrointestinal ulceration. They are recommended for use in patients at risk for gastrointestinal complications, such as those with prior peptic ulcer disease. The risk for ulcers can also be reduced by combining a nonselective NSAID with a second drug (eg, proton pump inhibitors, misoprostol) to reduce gastric acid secretion. Both COX 1 and COX 2 are expressed in renal tissues. Therapy with either nonselective NSAIDs or selective COX 2 inhibitors can cause fluid retention, aggravation of hypertension, and impaired renal function.

What is porphyria cutanea tarda?

Defects in uroporphyrinogen decarboxylase cause porphyria cutanea tarda (PCT), the most common form of porphyria. Patients with PCT exhibit chronic photosensitivity with blistering in areas of sun exposure and elevated levels of uroporphyrinogen in the urine. - painless blisters - increased skin fragility on the dorsal surfaces of the hands, facial hypertrichosis and hyperpigmentation - triggered by ingestion of certain substances (ethanol, estrogens) -Associated with Hep C -and mixed cryoglobulinemic vasculitis, which presents with nonblanching palpable purpura. DX: elevated urinary porphyrin levels TX: phlebotomy or hydroxychloroquine may provide relief, interferon-alpha (esp in pts w/ Hep C)

In a patient with delirum secondary to some cause, would you prescribe after tx of the original cause to address behavioral changes?

Delirium is an acute-onset "confusional state" characterized primarily by waxing and waning mental status changes and impaired attention. Disorientation, agitation, psychosis, and sleep disturbances may also occur. Delirium occurs secondary to an underlying medical condition, such as a urinary tract infection, and therefore the primary management is treating the underlying cause. The elderly and those with preexisting cognitive disorders are at a higher risk for delirium and may present with varying degrees of agitation. High-potency, first-generation antipsychotics (eg, haloperidol) and some second-generation antipsychotics (eg, quetiapine) can be used for the acute treatment of agitation and psychosis associated with delirium. Antipsychotic use is appropriate in the treatment of delirium in the elderly if the patient is at risk of acute harm to self or others and behavioral interventions have failed. Under these conditions, the benefits of antipsychotics (ie, the provision of safety) outweigh the potential risks when used at low doses and short durations.

A 48-year-old previously healthy woman comes to the office due to progressively worsening muscle weakness for the past 2 months. The patient has difficulty with activities such as climbing stairs, getting up from chairs, and placing dishes in overhead cabinets. She also reports a 4.5-kg (10-lb) unintentional weight loss and occasional abdominal discomfort over the same interval. Physical examination shows weakness of the shoulder and hip girdle muscles. She also has erythematous plaques on her joints. Further evaluation of this patient is most likely to reveal which of the following associated conditions?

Dermatomyositis is a systemic autoimmune disease characterized by proximal muscle weakness resembling polymyositis, with additional inflammatory features involving the skin. Specific skin findings include the heliotrope rash in the periorbital area and cheeks and Gottron papules, which are raised erythematous plaques over the joints and bony prominences of the hands (as seen in this patient). Muscle biopsy is diagnostic and shows mononuclear perifascicular inflammation and atrophy (ie, occurring along the periphery of the fascicles). Dermatomyositis may occur alone or as a paraneoplastic syndrome of an underlying malignancy, most commonly due to underlying adenocarcinoma (eg, ovary, lung, pancreas). Symptoms may precede the diagnosis of malignancy but often parallel the course of the cancer.

A 64-year-old man comes to the office due to slowing of his movements and gait instability that has been the cause of several recent falls. The patient was diagnosed with Parkinson disease a year ago. On examination, there is a resting tremor involving the right hand and rigidity during passive flexion and extension of the right upper limb. The patient is started on therapy with pramipexole. Which of the following best describes the mechanism of action of this medication?

Dopamine agonists have a chemical structure similar to the neurotransmitter dopamine and directly stimulate dopamine receptors. They do not have to be metabolized to be active. There are 2 classes of dopamine agonists: 1. Ergot compounds (derived from ergot fungi): bromocriptine 2. Nonergot compounds: pramipexole and ropinirole Dopamine agonists have an important role in the treatment of Parkinson disease (PD) as these medications have a long half-life and can delay the need to start levodopa, thereby postponing the development of motor fluctuations until later in the disease course. Bromocriptine also treats hyperprolactinemia.

What are ephelides and solar lentigines

Ephelides (eg, freckles) and solar lentigines (in pic) are found on sun-exposed areas. Ephelides are caused by increased melanin production by melanocytes, whereas solar lentigo represents an increased proliferation of melanocytes themselves. They are both hyperpigmented and flat, but solar lentigines are often larger and occur in adults.

A 46-year-old man is admitted to the hospital for atypical chest pain. His medical history is significant for hypertension controlled with amlodipine and hypercholesterolemia treated with atorvastatin; he has a family history of depression and heart disease. During review of his substance use history, the patient says that he has been a "regular drinker" for the last 5 years. He drinks a 6-pack of beer every night and has 2 more beers in the morning to help him "get through the day." He also admits to smoking marijuana occasionally but does not use tobacco or other illicit drugs. Which of the following symptoms or signs is most likely to appear earliest during this patient's hospitalization?

Ethanol use leads to a number of biochemical changes in the CNS. It acutely potentiates the effects of GABA (the primary inhibitory neurotransmitter in the CNS) at GABA-A receptors, leading to sedation. Chronic ethanol use causes downregulation of GABA receptors. Alcohol also weakly inhibits excitatory NMDA receptors in the brain, and chronic exposure leads to upregulation of these receptors. These adaptive changes result in tolerance (ie, the need to increase the dose to achieve the desirable effect) and symptoms of withdrawal on abrupt alcohol cessation, a common occurrence during hospital admission. Both tolerance and withdrawal are signs of substance dependence. Withdrawal symptoms can begin as early as 6 hours after the patient's last drink, typically reach maximal intensity in 2-3 days, and subside in 4-5 days if the withdrawal is not severe. Tremor, or "the shakes," is the most common initial finding. Patients also typically have signs of mild autonomic dysfunction (eg, increased heart rate and respirations), gastrointestinal distress (eg, nausea, vomiting), and anxiety.

Ewing sarcoma occurs in what part of the bone?

Ewing sarcoma typically arises in diaphysis of the long bones, especially the femur. It is the second most common malignant bone tumor in children after osteosarcoma. However, it occurs more frequently in older children (ages 10-15 years). Additionally, this patient's acute symptom onset and high-grade fever are more typical of an infectious process.

The patient was diagnosed with schizophrenia and initially treated with chlorpromazine. He now is reluctant to take the medication, saying, "I don't like how it makes me feel." His medication is subsequently changed to fluphenazine. Compared to the patient's initial treatment, fluphenazine is more likely to cause which of the following?

First-generation antipsychotics (FGAs), also known as typical or conventional antipsychotics, are effective in treating schizoph renia and psychosis due to other causes. FGAs are classified according to potency, which refers to the dose required to produce an effect. Low-potency FGAs primarily cause histaminergic, cholinergic, and noradrenergic blockade side effects. These drugs are highly sedating and often cause anticholinergic side effects and orthostatic hypotension. High-potency FGAs are associated primarily with extrapyramidal symptoms (eg, dystonia, akathisia, parkinsonism) due to potent D2 dopamine receptor antagonism in the nigrostriatal pathway. This patient's medication was changed from a low-potency FGA (chlorpromazine) to a high-potency FGA (fluphenazine). Muscular rigidity, an extrapyramidal side effect, is seen in drug-induced parkinsonism; it is more likely to occur with a high-potency agent.

A 3-year-old girl is brought to her pediatrician for a well child checkup. She has met all of the appropriate developmental milestones. Her height corresponds to the 60th percentile. Osteoblasts near the growth plates of her long bones secrete matrix material, and when they become trapped in the ossified matrix, they become known as osteocytes. These osteocytes remain connected to each other by:

GAP JUNCTIONS Within a single Haversian system, the central canal is encircled by multiple concentric lamellae of bony matrix that each contains lacunae filled with osteocytes and extracellular bone fluid. Delicate canaliculi radiate from each lacuna to create a reticular network with adjacent lacunae, and the cytoplasmic processes of the osteocytes lie within these canaliculi. These cytoplasmic processes send signals to and exchange nutrients and waste products with the osteocytes within neighboring lamellae via gap junctions. The osteocytes serve to maintain the structure of the mineralized matrix and control the short-term release and deposition of calcium (i.e., calcium homeostasis). The plasma calcium concentration directly dictates the metabolic activity of osteocytes, while parathyroid hormone and calcitonin indirectly influence their metabolic activity. Osteocytes can also sense mechanical stresses and send signals to modulate the activity of surface osteoblasts, thereby helping to regulate bony remodeling.

A 45-year-old woman comes to the office due to gradually progressive pain, stiffness, and swelling of her hand joints for the past several months. The patient has had prolonged morning stiffness and significant restriction of her daily activities due to the pain, as well as generalized fatigue. She describes the symptoms over the last several weeks as "disabling," and she needs assistance with activities of daily living in the morning. The patient has tried over-the-counter pain medications, such as acetaminophen and ibuprofen, with only minimal relief of her symptoms. She has no other medical problems. Family history is significant for diabetes mellitus on her maternal side. Which of the following drugs would provide the most rapid relief of her symptoms?

GCs This patient has swelling, pain, and morning stiffness in multiple joints for >6 weeks, consistent with rheumatoid arthritis (RA). The foundation of management for RA is disease-modifying antirheumatic drugs (DMARDs), which alleviate pain and inflammation and reduce long-term joint destruction and disability. Examples of DMARDS are methotrexate (typically first-line), sulfasalazine, hydroxychloroquine, minocycline, and tumor necrosis factor-alpha inhibitors. However, the response to DMARD therapy typically takes weeks. therefore, short-term treatment with anti-inflammatory therapies, including systemic and intraarticular glucocorticoids (eg, prednisone) or nonsteroidal anti-inflammatory drugs (NSAIDs), can provide rapid, temporary relief of symptoms in patients starting on DMARDs. However, they do not provide adequate long-term control of disease or prevention of joint deformity. Glucocorticoids exert their anti-inflammatory effects by inhibiting phospholipase A2, which decreases prostaglandin and leukotriene synthesis. Glucocorticoids also depress the immune response by inhibiting the transcription of multiple cytokines and adhesion proteins, which reduces leukocyte recruitment and activation. FYI Colchicine is used in the management of both acute and chronic gout. Colchicine works by binding to tubulin and inhibiting microtubule polymerization. This results in disruption of neutrophil chemotaxis and phagocytosis, thereby reducing the inflammatory response to uric acid crystals. Colchicine is not effective in RA.

Clinical presentation of goodpastures

Goodpasture syndrome is sometimes called anti-glomerular basement membrane (GBM) antibody disease. Presenting clinical manifestations of Goodpasture syndrome are hemoptysis, radiographic focal pulmonary consolidations, and a glomerulonephritis that may rapidly progress to renal failure.

She is interested in medication for her alcohol use disorder, but lives alone and is worried that her cravings will overwhelm her self control. A medication with which of the following mechanisms of action would be most effective in this patient?

In addition to psychosocial interventions (eg, motivational interviewing, Alcoholics Anonymous), several medications are available to treat alcohol use disorder, including naltrexone and acamprosate. Naltrexone blocks the mu-opioid receptor and inhibits the rewarding and reinforcing effects of alcohol, helping to reduce cravings and improving motivation to quit. The long-acting depot form (given as monthly injections) is useful for patients at risk for nonadherence with daily administration. Acamprosate, another abstinence-promoting drug, works by modulating glutamate neurotransmission at the N-methyl-D-aspartate receptor. The aldehyde dehydrogenase inhibitor disulfiram is considered a second-line treatment for alcohol use disorder because of poorer efficacy (effectiveness depends on supervised administration). When alcohol is ingested by a patient taking disulfiram, acetaldehyde accumulates and causes unpleasant adverse effects (eg, nausea, vomiting, flushing, sweating, headache).

In response to increasing estrogen levels, the myometrial cells start to express genes that encode connexin-43 and the oxytocin receptor. These molecular changes would result in increased formation of which of the following?

Initiation of labor requires a series of biochemical changes in the uterus and cervix mediated by endocrine changes. Cell junctions are multiprotein complexes that allow contact between adjacent cells or between a cell and the extracellular matrix. Communicating (gap) junctions are especially important during labor and delivery, which require coordination and synchronization of individual myometrial cells. Gap junctions permit diffusion of molecules between neighboring cellular cytoplasms through a connexon (cylinder with a central channel composed of 6 connexin proteins). Immediately prior to delivery, estrogen stimulates upregulation of gap junctions between individual myometrial smooth muscle cells. An increase in gap junction density at delivery heightens myometrial excitability. Gap junctions consist of aggregated connexin proteins (eg, connexin-43) that allow passage of ions between myometrial cells. Estrogen also increases expression of uterotonic (eg, oxytocin) receptors, which mediate calcium transport through ligand-activated calcium channels. The combination of an increase in gap junction density and uterotonic receptors results in coordinated, synchronous labor contractions.

What is the role of osteoprotegrin? What is Denosumab?

Interaction between the receptor for activated nuclear factor kappa B (RANK) and its ligand (RANK-L), is critical for the development of mature, multinucleated osteoclasts. This binding is blocked by osteoprotegerin (OPG), which acts as a decoy receptor for RANK-L (preventing it from interacting with RANK). By binding RANK-L, OPG reduces the differentiation and survival of osteoclasts, resulting in decreased bone resorption and increased bone density. Estrogen maintains bone mass in premenopausal women by inducing production of OPG by osteoblasts and stromal cells and decreasing expression of RANK on osteoclast precursors. The loss of OPG at menopause leads to increased osteoclast activity that predisposes to osteoporosis. Denosumab is a monoclonal antibody used in the treatment of postmenopausal osteoporosis. It works similar to OPG in that it binds RANK-L and prevents its interaction with RANK receptor. Denosumab therefore causes decreased osteoclast differentiation and activity as well as decreased bone resorption.

HIV patient. Motor strength in the right upper and lower extremities is decreased, and his gait is ataxic. Brain MRI reveals several discrete areas of demyelination in the subcortical and periventricular white matter with no surrounding edema or mass effect. CD4 cell count is 30/mm3. Reactivation of which of the following pathogens is most likely responsible for this patient's current symptoms?

JC virus is a polyomavirus, typically acquired in childhood, that causes life-long, latent infection of the kidney and lymphoid organs. Most adults have serologic evidence of exposure but remain asymptomatic. However, those with severe immunosuppression, particularly patients with advanced AIDS, are at risk for viral reactivation with spread to the brain. The virus attacks oligodendrocytes, the cells that produce myelin, causing a severe demyelinating disease called progressive multifocal leukoencephalopathy (PML). PML typically presents with slowly worsening confusion, ataxia, motor deficits, and seizure. Brain MRI usually shows multifocal areas of white matter demyelination with no mass effect or enhancement. Diagnosis is confirmed with JC virus PCR testing of the cerebrospinal fluid; brain biopsy is rarely required. There is no specific treatment for PML, but patients with HIV who develop the disease are treated with antiretroviral therapy to reduce immune suppression. This often helps stabilize neurologic symptoms.

A 45-year-old woman with a history of opioid use disorder and HIV infection is brought to the emergency department due to worsening lethargy. The patient receives methadone maintenance therapy, which she recently restarted after being hospitalized for an HIV-related infection. Her infection improved after inpatient treatment, and she continues to take appropriate antimicrobials as an outpatient. Her current temperature is 37 C (98.6 F), blood pressure is 116/68 mm Hg, pulse is 90/min, and respirations are 10/min. On physical examination, the patient is somnolent and withdraws all extremities to painful stimuli. The pupils are small and sluggish to react. Brain imaging reveals no new findings, and renal function tests are within normal limits. Use of which of the following medications most likely contributed to this patient's current condition?

Methadone is a mu-opioid receptor agonist used for maintenance treatment of opioid use disorder. It has a long half-life, which allows it to effectively suppress cravings and withdrawal symptoms. Like all opioid medications, methadone carries a risk of overdose, which is characterized by somnolence, miosis, and respiratory depression (ie, bradypnea). Another side effect is QTc prolongation, which can predispose to torsade de pointes. Methadone is extensively metabolized by the cytochrome P-450 system, particularly by CYP3A4. In this case, the patient likely received fluconazole, an antifungal medication that is used in the treatment of cryptococcal meningitis (which typically occurs in patients with a CD4 count <100 cells/µL). Fluconazole inhibits CYP3A4, which can increase plasma methadone concentration and lead to opioid toxicity. Interestingly, not all P-450 3A4 inhibitors prolong methadone effects; other medications known to cause similar interactions with methadone include voriconazole, ketoconazole, ciprofloxacin, clarithromycin, cimetidine, and fluvoxamine.

32-year-old woman complains of weakness in her hands and "heaviness" in her eyelids at the end of each day. Chest imaging shows an anterior mediastinal mass. The organ from which this mass most likely originated shares its embryologic origin with:

Myasthenia gravis causes muscle weakness, with the extraocular muscles most commonly affected. Patients often experience ptosis and diplopia. The muscle weakness worsens with activity, and patients often note that their symptoms are worse at the end of the day. The majority of patients with myasthenia gravis are found to have a thymoma or thymic hyperplasia. The thymus is derived from the third pharyngeal pouch, as are the inferior parathyroid glands. The table below lists the derivatives of the pharyngeal pouches, grooves and membranes:

Differentiate Myasthenia Gravis from Lambert Eaton myasthenic syndrome

Myasthenia gravis is caused by anti-acetylcholine receptor autoantibodies and is characterized by episodic weakness that initially affects the ocular/bulbar musculature. The Eaton-Lambert myasthenic syndrome is a paraneoplastic condition causing fatigable weakness in the extremities; it is caused by autoantibodies to presynaptic calcium channels. Light microscopy of muscle biopsy specimens is normal in these conditions.

A 43-year-old man is hospitalized with recent-onset oliguria and a high serum creatinine level. He has been seen in clinic several times for an intranasal ulcer that has failed to heal. This patient's condition is most likely associated with antibodies against which of the following?

Nasal mucosal ulcerations and glomerulonephritis are most characteristic of granulomatosis with polyangiitis (Wegener). Cytoplasmic-staining antineutrophil cytoplasmic antibodies (c-ANCAs) are virtually pathognomonic for granulomatosis with polyangiitis, with a better than 90% specificity and sensitivity. Proteinase 3-ANCA (also known as c-ANCA) may also be useful as a quantitative measure of disease activity.

Explain how negative predictive value varies with prevalence/pretest probability

Negative predictive value (NPV) is defined as the probability of not having a disease when the test result is negative. NPV is calculated as the proportion of true negatives divided by the total number of negative tests (true and false negatives); therefore, it varies with the prevalence of disease in the target population. The prevalence of a disease in a population may be used as an estimate for the pretest probability of having the disease in patients who closely resemble the population. A patient with a high pretest probability will have a low NPV with a negative test, whereas a patient with a low pretest probability will have a high NPV with a negative test. This patient has a high pretest probability for having thyroid cancer since she most likely received significant radiation exposure to the thyroid during childhood. As such, her negative cytology results would have a lower NPV when compared to another patient with no risk factors for thyroid malignancy who also tests negative.

The patient undergoes amniocentesis during the 18th week of pregnancy. Amniotic fluid analysis shows an increased level of acetylcholinesterase. This patient's amniocentesis results most likely suggest failure of which of the following processes?

Neural tube defects (NTDs) occur due to failure of fusion of the neural plate edges during the 4th week of fetal development. Fusion begins in the cervical region and proceeds toward the cranial and caudal ends of the neural tube (ie, the rostral and caudal neuropores). Failure of the rostral neuropore to close results in anencephaly, whereas impaired closure of the caudal neuropore causes spina bifida. If either neuropore does not fuse, an opening persists between the neural tube and the amniotic cavity that allows leakage of alpha-fetoprotein (AFP) and acetylcholinesterase (AChE) into the amniotic fluid. AFP can also cross the placenta and is detectable in the maternal serum. Detection of AFP and AChE is used for prenatal screening of NTDs (eg, spina bifida cystica, anencephaly).

Other forms of hydrocephalus are

Other forms of hydrocephalus are described below: Communicating hydrocephalus presents with global ventricular dilation and increased intracranial pressure (ICP) without significant blockage to CSF flow within the brain or brainstem. It usually results from impaired CSF absorption by the arachnoid villi due to inflammation and fibrosis caused by hemorrhage or infection (Choice A). It rarely occurs due to increased production of CSF caused by papillomas of the choroid plexus. Normal pressure hydrocephalus presents with enlarged ventricles in the setting of normal ICP. It is considered to be a form of communicating hydrocephalus where impaired absorption of CSF is compensated for by decreased CSF formation. Although ICP is normal, there is still a pressure gradient between the ventricles and subarachnoid space which is responsible for the ventricular enlargement. Communicating hydrocephalus is more often found in infants and children, while normal pressure hydrocephalus is most commonly found in adults. Hydrocephalus ex vacuo presents with normal ICP and increased dilatation of ventricles secondary to cortical atrophy. It occurs most often in elderly patients with dementia. Pseudotumor cerebri presents with elevated ICP and classically occurs in young women who are overweight. The etiology of the condition is not well known, but it is thought to be related to cerebral venous outflow abnormalities due to elevated intracranial pressures.

Patient who is going crazy in the ED has nystagmus. What substance was he likley on?

PCP FYI this chart shows symptoms with the drug This patient's combative behavior, confusion, poor judgment, memory loss, and hallucinations, accompanied by nystagmus and ataxia on physical examination, are consistent with phencyclidine (PCP) intoxication. PCP is a hallucinogen that works primarily as an N-methyl-D-aspartate glutamate receptor antagonist, as well as a monoamine reuptake inhibitor. PCP was originally developed as an anesthetic, but its use was discontinued due to its adverse effects in humans. Moderate amounts of PCP cause dissociative symptoms, including detachment and withdrawal. At higher doses, it can induce agitation, hallucinations, and violent behavior. Ataxia and nystagmus (both horizontal and vertical) are other distinguishing symptoms of PCP abuse. Hospital urine toxicology screens usually test for PCP.

For a patient with panic disorder, what is good long term pharmacotherapy?

Panic disorder is a chronic anxiety disorder characterized by recurrent, unexpected panic attacks, anticipatory anxiety about future attacks, and phobic avoidance. Both antidepressants and benzodiazepines are effective for panic disorder. Among the antidepressants, selective serotonin reuptake inhibitors (SSRIs) and serotonin-norepinephrine reuptake inhibitors are preferred first-line therapy due to their relatively benign side effect profile compared to tricyclic antidepressants and monoamine oxidase inhibitors (MAOIs). Benzodiazepines are also effective and have more rapid onset of action. They are useful in severely symptomatic and functionally impaired patients who require rapid relief. However, benzodiazepines carry risks for abuse and result in physiological dependence and withdrawal if stopped abruptly. Benzodiazepines should be avoided in patients with a history of substance abuse. An SSRI would be the preferred choice in this patient who is not acutely symptomatic and has a history of alcohol abuse

Types of encephalitis in HIV patients

Patients with advanced AIDS can occasionally develop cytomegalovirus encephalitis, which usually presents with rapidly progressive altered mental state and focal neurologic deficits. However, this condition is highly inflammatory so brain imaging typically shows enhancing lesions. Epstein-Barr virus reactivation can cause primary CNS lymphoma in patients with advanced AIDS. Patients usually have focal neurologic symptoms and altered mental state. However, brain MRI normally reveals a single enhancing lesion with mass effect. Herpes simplex virus encephalitis usually causes focal neurologic deficits and altered mental status. However, symptom onset is usually rapid (<1 week), most patients have fever, and brain imaging usually reveals an enhancing temporal lobe lesion with mass effect. Measles can cause subacute, sclerosing panencephalitis several years after initial infection. This fatal, progressive disease is characterized by slowly worsening neurologic symptoms and multifocal, enhancing, white matter lesions. Toxoplasma gondii is usually acquired by exposure to cat feces. Most patients are asymptomatic and have a life-long dormant infection. However, patients with advanced AIDS can have reactivation leading to meningoencephalitis (eg, headache, confusion, fever). Brain imaging usually shows several ring-enhancing lesions with surrounding edema.

A novel antimicrobial agent that inhibits bacterial glycosyltransferase enzyme is developed. Bacteria grown in a medium containing this antibiotic become spherical, cease to grow, and subsequently lyse. So that the activity of this antibiotic against various bacteria can be determined, disks enriched with the drug are placed on agar plates containing bacterial colonies. The plates are incubated alongside control plates with the same organisms but no antimicrobial disks. The control plates all show organism growth. In the plates with the diffusion disks, resistance to the drug is determined by measuring the zone of complete growth inhibition around the disk. Which of the following bacteria is most likely to be resistant to this drug?

Peptidoglycan is the main component of the bacterial wall in both gram-positive and gram-negative organisms. It is composed of a linear glycan chain of 2 alternating sugars, N-acetylglucosamine and N-acetylmuramic acid, that are cross-linked by short peptides, thereby forming a rigid matrix. The enzyme glycosyltransferase is a crucial component of peptidoglycan synthesis; it adds glycan molecules to the growing peptidoglycan chain. Inhibition of this enzyme would result in gaps in the bacterial cell wall, with subsequent loss of bacterial shape and cell lysis from osmotic stress. Although most bacteria have a cell wall composed of peptidoglycan, organisms from the Mycoplasma genus (eg, Ureaplasma urealyticum, M hominis) completely lack a cell wall. These pathogens are separated from the environment by a single phospholipid bilayer membrane composed primarily of cholesterol (similar to human cells). Therefore, antibacterial agents that target the cell wall, including penicillins, cephalosporins, carbapenems, vancomycin, and the drug described in this study, would be ineffective. Mycoplasma is treated primarily by medications that inhibit bacterial ribosomal function such as macrolides and tetracyclines.

A 45-year-old man with a history of chronic alcohol use disorder is brought to the emergency room due to altered mental status. The patient appears malnourished. He is given thiamine, folic acid, a multivitamin, and dextrose-containing intravenous fluids. However, the patient develops marked muscle weakness a few hours later. Laboratory studies reveal a serum phosphate concentration of 0.5 mg/dL (normal: 2.5-4.5). Which of the following is the most likely cause of this patient's low serum phosphate level?

Phosphorus is involved in multiple biologic processes, including cellular energy metabolism, bone formation, and acid-base homeostasis. Although biologically active phosphorus is largely found intracellularly, serum phosphorus levels are often reflective of available body stores and are maintained through the action of hormones (eg, parathyroid hormone, calcitriol, FGF-23) on the small intestines, bones, and kidneys. Malnourishment (eg, due to chronic alcohol use disorder) results in the depletion of phosphate, although serum levels may remain normal due to transcellular shifts. Reintroduction of carbohydrates (ie, dextrose-containing intravenous fluids) increases insulin secretion, which stimulates the redistribution of phosphate from the serum into muscle and hepatic cells for use during glycolysis (eg, formation of ATP, 2-3 diphosphoglycerate). This leads to profound hypophosphatemia; lack of adequate intracellular phosphate can result in failure of cellular energy metabolism, producing the clinical features of refeeding syndrome (eg, muscular weakness, arrhythmias, congestive heart failure).

Physical examination reveals tachycardia, distended neck veins, bilateral crackles on lung auscultation, a third heart sound on cardiac auscultation, and pedal edema. The appropriate therapy is initiated. On the third day of hospitalization, her hematocrit level increases to 50%, up from 44% on admission. An arterial blood gas analysis shows an arterial O2 partial pressure (PaO2) of 70 mm Hg. A 51Cr-tagged red blood cell infusion indicates normal red blood cell mass. Which of the following is the most likely cause of this patient's polycythemia?

Polycythemia (erythrocytosis) is defined as a hematocrit level > 52% in men and > 48% in women. It may be the result of a true increase in the red blood cell (RBC) mass (absolute erythrocytosis) or decrease in the plasma volume (relative erythrocytosis). In a patient with erythrocytosis, the following tests can help determine the cause: 1. Absolute versus relative erythrocytosis: Hemoglobin and hematocrit levels cannot accurately differentiate these conditions. Direct measurement of the RBC mass is necessary. An increased total RBC mass indicates an absolute erythrocytosis; a normal RBC mass indicates a relative erythrocytosis. This patient does not have an increased RBC mass and is likely experiencing relative erythrocytosis due to aggressive diuretic therapy for her heart failure exacerbation. 2. Primary versus secondary erythrocytosis: Serum erythropoietin levels can be used to differentiate primary from secondary erythrocytosis. Primary erythrocytosis is associated with low erythropoietin levels and is caused by myeloproliferative disorders such as polycythemia vera. Secondary erythrocytosis is characterized by increased erythropoietin levels due to chronic hypoxia from high altitudes, smoking, or chronic obstructive pulmonary disease; or abnormal secretion by neoplastic or otherwise diseased tissues. 3. Hypoxic versus other causes of secondary erythrocytosis: Measurement of the arterial oxygen saturation (SaO2) is important to exclude hypoxemia as a cause of the erythrocytosis. SaO2 < 92% (PaO2 < 65 mm Hg) can cause secondary polycythemia. SaO2 (measured as a percentage) should not be confused with PaO2 (measured as a partial pressure in mm Hg).

What is primary billiary cholangitis

Primary biliary cholangitis (previously primary biliary cirrhosis) is an autoimmune liver disease that can present with cholestasis, malabsorption, pruritus, and jaundice. Although the risk of this condition is increased in patients with autoimmune disease (eg, CREST syndrome, Sjögren syndrome),

Loss of only proprioceptive function is seen with damage to ____ in what condition?

Selective loss of proprioceptive (along with vibratory and light touch) sensation occurs with isolated injury of the bilateral dorsal columns. A classic example is tabes dorsalis, a form of tertiary neurosyphilis that causes damage to the dorsal columns and dorsal nerve roots.

Describe K+ journey thru the kidney

Several segments of the nephron are involved in the management of potassium (K+). However, most handle K+ at a relatively fixed rate that is independent of potassium load and do not play a significant role in the regulation of K+ excretion in the urine. These segments include the following: -Bowman's capsule: Because K+ is freely filtered across the glomerular membrane, the amount of K+ within Bowman's space is equal to that in the glomerular capillaries (ie, 100%) -The proximal tubule: Approximately 65% of the filtered K+ load is reabsorbed in the proximal tubule, leaving ~35% of the total filtered load. -The thick ascending limb of the loop of Henle: Further resorbs about 25%-30% of the filtered K+ load through the action of the Na+/K+/2Cl- cotransporter, resulting in only 5-10% of K+ remaining in the tubular fluid after this segment Because this is a stable process, even in hyperkalemic states, patients will reabsorb the majority of filtered K+ in the proximal tubule and loop of Henle. Potassium regulation is therefore primarily mediated by the principal and α-intercalated cells of the late distal and cortical collecting tubules. -Hypokalemia stimulates reabsorption of K+ via apically located H+/K+-ATPases on α-intercalated cells and can cause the amount of K+ in the collecting tubule to approach 1% of the filtered load. Conversely, an increased K+ load stimulates principal cells to secrete K+ through apical K+ channels. High dietary K+ intake can cause the amount of K+ in the collecting tubules to actually exceed the filtered load (ie, >100%). -Excessive K+ intake increases K+ excretion through the following mechanisms: Hyperkalemia stimulate basolateral Na+/K+ pumps on principal cells, increasing K+ secretion into the tubular fluid. Elevated K+ levels also increase aldosterone secretion, which further enhances activity of principal cell Na+/K+ pumps and also increases their apical permeability to Na+ and K+ (leading to K+ loss in the tubular fluid).

A 7-month-old boy is brought to the physician by his parents due to irritability and white patches in his mouth. His past medical history is significant for 3 episodes of otitis media and 2 episodes of bronchiolitis that have required hospitalization. He also has a history of chronic loose stools. The child is small for his age and ill-appearing. Head and neck examination shows white patches consistent with oral candidiasis but is otherwise normal. Auscultation of the lungs shows expiratory wheezing. Cardiac examination is within normal limits. Serum protein electrophoresis shows a very low gamma globulin level. Chest x-ray reveals an absent thymic shadow. Which of the following is the most likely diagnosis?

Severe combined immune deficiency (SCID) is a life-threatening immunodeficiency syndrome that presents in infancy. It is caused by a variety of mutations in different genes that result in impaired T and B cell development and function. This leads to compromised cell-mediated and humoral immunity with the eventual development of severe viral and bacterial infections as maternal immunity wanes. Other common features include mucocutaneous candidiasis, persistent diarrhea, and failure to thrive. Laboratory studies show very low or absent CD3+ T cells and hypogammaglobulinemia. Thymic hypoplasia or aplasia is another common finding in infants with SCID due to severe T cell deficiency. DiGeorge syndrome (DGS) classically presents with craniofacial abnormalities, conotruncal cardiac anomalies, hypoplastic thymus, and hypocalcemia resulting from parathyroid hypoplasia. Immunodeficiency can also occur and is related to the degree of thymic hypoplasia. Although complete DGS (<1% of cases) is a form of SCID, the absence of other characteristic findings makes DGS a less likely diagnosis. In Wiskott-Aldrich syndrome, functioning of both T and B cells is impaired due to abnormal cytoskeletal functioning. Patients experience recurrent viral, bacterial, and fungal infections that generally worsen with age. However, Wiskott-Aldrich syndrome also typically causes bleeding due to thrombocytopenia and eczema.

Damage to temporal lobe results in what

Temporal cortex injury can cause disturbances in language, sensory interpretation, and impaired memory. These patients can also exhibit behavioral changes, such as apathy, hyperorality, hypersexuality, and visual agnosia as seen in Klüver-Bucy syndrome. Nondominant (usually right-sided) lesions can affect nonverbal memory, including musical ability; dominant left-sided lesions can affect verbal memory, such as word recognition.

What region of the skeletal muscle sarcomere contains only thick filaments

The H band is the region of the sarcomere containing only thick (myosin) filaments. On electron microscopy, the H band is a portion of the A band that straddles the M line. The A band corresponds to the thick filaments in the sarcomere and includes portions overlapped by thin (actin) filaments During muscle contraction, the thin filaments slide over the thick filaments toward the M line, reducing the length of the H band. The A band always remains the same length. he Z line is typically distinctly darker than the remainder of the sarcomere. Thin filaments, which are composed of actin, tropomyosin, and troponin, anchor at the Z line. The segment of the sarcomere between the H and I bands is where thick and thin filaments overlap. The I band contains the Z line and only those sections of the thin filaments that do not overlap with thick filaments. During muscle contraction, both the I band and H band decrease in length.

Damage to the arcuate fasiculus causes what?

The arcuate fasciculus is a neural pathway that connects the Broca and Wernicke areas, which are responsible for expressive and receptive language, respectively. Disruption of the arcuate fasciculus classically results in conduction aphasia, characterized by fluent speech, intact comprehension, and impaired repetition.

Overuse of the coracobrachialis muscle (bench pressing) can present with what

The coracobrachialis originates from the coracoid process of the scapula and acts to flex the arm at the glenohumeral joint. The musculocutaneous nerve pierces the muscle near its insertion into the humerus; overuse of the coracobrachialis (eg, excessive bench pressing) can cause entrapment of the musculocutaneous nerve, causing impaired elbow flexion and lateral forearm sensory loss.

What would be the consequence of lateral medullary infarction?

The lateral medullary spinothalamic tract transmits pain and temperature signals from the contralateral body to the thalamus. Lateral medullary infarction (Wallenberg syndrome) presents with loss of pain and temperature sensation on the contralateral body and ipsilateral face as well as vertigo, hoarseness, dysphagia, and abnormal eye movements.

Disruption of the hypothalamospinal tract results in what?

The hypothalamospinal tract projects from the hypothalamus to the ciliospinal center of the intermediolateral cell column (T1-L2), providing sympathetic innervation to the ipsilateral eye and face. Disruption of this tract typically results in ipsilateral Horner syndrome (eg, ptosis, miosis, anhidrosis)

Damage to the lateral prefrontal cortex results in what

The lateral prefrontal cortex is involved in executive functioning, which includes motivation, organization, planning, and purposeful action. Damage to the lateral prefrontal cortex may result in dysexecutive syndrome, a syndrome characterized by significant difficulties performing these functions.

Interference with the nigrostriatal pathway by D2 receptor blockers results in what?

The nigrostriatal pathway projects from the substantia nigra to the caudate nucleus and putamen and primarily regulates the coordination of voluntary movements. D2 receptor blockade in this pathway results in extrapyramidal effects (eg, dystonia, akathisia, tardive dyskinesia) and drug-induced parkinsonism.

Optic chiasm lesions result in what

The optic chiasm is superior to the cavernous sinus and is where the optic nerve fibers from the nasal side of each retina cross to the opposite side of the brain. Optic chiasm lesions typically cause bitemporal hemianopia,

Damage to the parietal lobe results in what

The parietal cortex processes and interprets visual, auditory, and motor signals received from other brain areas. Parietal cortex damage results in difficulties with spatial and visual perception. Dominant lesions (>95% of right-handed and >50% of left-handed patients are left hemisphere dominant) result in Gerstmann syndrome with right-left confusion and difficulty with writing and mathematics. Nondominant parietal cortex lesions (most commonly right-sided) can result in hemi-neglect, constructional apraxia, and denial of the problem.

Entrapment of the ________ nerve in the quadrangular space results in....

The teres major muscle and long head of the triceps brachii form the inferior and medial borders of the quadrangular space through which the axillary nerve passes. Entrapment of the axillary nerve within the quadrangular space can cause sensory loss over the lateral shoulder and impair shoulder abduction.

Describe the stages of sleep

Theta waves are seen in stage 1 (transition from wakefulness to sleep) and stage 2 (light, stable sleep). Sleep spindles and K complexes are characteristic of stage 2. Delta waves are characteristic of slow-wave sleep or deep sleep (stage 3). Non-REM arousal disorders (eg, sleep terrors) often occur in stage 3.

A 6-year-old boy is brought to the office due to frequent nighttime awakenings. Several times a week over the past month, he has come to his parents' bed around 3 AM, frequently crying, appearing frightened, and saying that he became lost in a crowd and couldn't find his parents. Although he is comforted by their reassurance, he refuses to return to his room and eventually falls back asleep in their bed. The sleep stage associated with this child's frightening experience is characterized by which of the following?

This boy's recurrent awakenings with associated dream content and ability to be comforted are characteristic of nightmares, a common and developmentally normal experience for most children. Nightmares occur during rapid eye movement (REM) sleep and are more frequent in the final third of the night when REM periods become longer. Children can often describe the nightmare, become fully alert after awakening, and are typically able to be consoled. In contrast, sleep terrors are a common non-REM parasomnia of childhood that occur during slow-wave sleep in the first third of the night. Unlike nightmares, the child with sleep terrors does not fully awaken, is unresponsive to comfort, and has no associated dream content. REM sleep is characterized by the defining feature of rapid eye movements, vivid dreaming, and voluntary muscle atonia due to inhibition of motor neurons. EEG activity during REM sleep is similar to the waking state, consisting of a combination of alpha, beta, and desynchronous EEG waves (sawtooth waves).

A 12-year-old girl is brought to the office by her mother due to unusual nighttime behavior. The mother reports that once or twice a month over the past year she has found her daughter walking around the house between the hours of midnight and 2 AM. The mother says, "Although her eyes are open, she appears to be in a daze and doesn't respond to me. I guide her back to bed and she goes back to sleep, but I worry that she could get hurt or fall down the stairs if I'm not there." The girl has no recollection of the events. Which of the following EEG patterns is characteristic of the sleep stage associated with this behavior?

This girl's nighttime ambulation associated with incomplete arousal and amnesia for the event is consistent with sleepwalking, a non-REM (NREM) parasomnia. Sleepwalking is more common in children, peaks at age 8-12, and often remits spontaneously. NREM-related parasomnias such as sleepwalking and sleep terrors occur during deep, slow-wave sleep. Deep, slow-wave sleep is more prominent during the first half of the night and, as a result, NREM-related parasomnias occur more commonly during this time. Sleep is divided into NREM and REM sleep. Most dreams occur during REM sleep. NREM sleep is further divided into 3 substages based on depth of sleep (N1, N2, and N3). Brain waves of different frequencies are associated with different stages of sleep. Delta waves (frequency up to 3.99 Hz) are characteristic of deep, slow-wave sleep (stage N3). Alpha and beta waves are characteristic of wakefulness and REM sleep (Choices A and B). K complexes and Sleep spindles are characteristic of stage N2 (light, stable sleep)

His temperature is 39.4 C (103 F), blood pressure is 130/80 mm Hg, pulse is 98/min, and respirations are 20/min. On examination, dullness to percussion, crackles, and egophony are present at the right lung base. The remainder of the examination is normal. Chest x-ray shows a dense infiltrate occupying the entire right lower lobe. Which of the following most likely accounts for the color of this patient's sputum?

This middle-aged man presenting with new-onset fevers, productive cough, and a dense lobar infiltrate likely has community-acquired pneumonia (CAP). CAP in otherwise healthy individuals is most commonly caused by Streptococcus pneumoniae, the most common bacterial etiology worldwide. Tobacco use further increases its risk. In nonelderly patients, pneumococcal pneumonia presents with abrupt-onset fevers, rigors, tachypnea, and productive cough with consolidation in one lobe of the lungs. Neutrophil myeloperoxidase is responsible for the green color of pus and sputum in bacterial infections. It is a blue-green heme-based pigmented molecule contained within the azurophilic granules of neutrophils and catalyzes the production of hypochlorous acid from chloride and hydrogen peroxide during the phagocytic respiratory burst.

24-hour urinary cortisol is increased. Further evaluation reveals that high-dose, but not low-dose, dexamethasone suppresses serum cortisol levels. Serum ACTH levels are high-normal. Which of the following is the most likely cause of this patient's symptoms?

This patient has weight gain, proximal muscle weakness, hypertension, and hyperglycemia, which are prominent features of Cushing syndrome. Screening tests for endogenous Cushing syndrome include low-dose dexamethasone suppression testing. In normal individuals, low doses of dexamethasone (a potent glucocorticoid) suppress ACTH and cortisol levels through negative feedback. However, in endogenous Cushing syndrome, ACTH and cortisol levels are not suppressed. The most common causes of Cushing syndrome are shown in the flowchart and can be categorized on the basis of serum ACTH levels as either ACTH dependent (ACTH is high or inappropriately normal) or ACTH independent (ACTH is suppressed). In ACTH-dependent Cushing syndrome, if high-dose dexamethasone suppresses ACTH and cortisol secretion, a pituitary source of ACTH (ie, ACTH-secreting pituitary adenoma) is likely. In contrast, ectopic ACTH production (eg, from malignant tumors such as small cell lung cancer) is not suppressed by high-dose dexamethasone This patient has excessive cortisol production that is suppressed by high-dose but not low-dose dexamethasone. This is consistent with an ACTH-secreting corticotroph pituitary adenoma (Cushing disease), the most common cause of endogenous Cushing syndrome.

A 23-year-old man is evaluated for excessive bruising since childhood. He says some of his family members "bleed easily" and sometimes require blood transfusions. Temperature is 36.7 C (98.1 F). Physical examination is unremarkable. Complete blood cell count is unremarkable. Platelet aggregometry is abnormal. Genetic testing identifies a defect in a platelet surface glycoprotein receptor that normally binds fibrinogen to support platelet aggregation. The abnormal glycoprotein receptor found in this patient is most likely targeted by which of the following medications?

This patient likely has Glanzmann thrombasthenia, an autosomal recessive disorder that is caused by a deficient or defective glycoprotein (GP) IIb/IIIa on platelet surfaces and that typically presents in childhood with mucocutaneous bleeding. Peripheral smear shows no platelet clumping (an important clue for diagnosis). Platelets are responsible for formation of platelet plugs that stop bleeding from injured vessels (primary hemostasis). Vessel wall injury exposes the subendothelial collagen and matrix. Platelet attachment to exposed collagen is strengthened by GP Ib binding to von Willebrand factor on the vessel wall. The resulting platelet activation leads to the following: Release of mediators (eg, ADP, thromboxane A2 [TXA2]) into circulation, which in turn activates other platelets Conformational structural change of GP IIb/IIIa on platelet surfaces; this allows thousands of copies of GP IIb/IIIa to bind fibrinogen, thereby forming a platelet plug. Abciximab, a GP IIb/IIIa receptor antagonist, inhibits binding of this receptor to fibrinogen. Abciximab and other GP IIb/IIIa inhibitors are useful for treatment of unstable angina and acute coronary syndrome, particularly in patients undergoing percutaneous coronary intervention.

Patient comes in to the ED and says she feels like she is dying. The pain resolves within 10 minutes without treatment, and the patient reports, "I was taking the bus home from work when my chest started feeling really tight. I'm lucky my friend was there and able to help me get to the hospital. What if she's not there next time?" She describes experiencing similar episodes at random places and times that are characterized by a pounding heart, trembling, dizziness, and sweating. She drinks alcohol socially and does not use any medications. This patient is most likely to develop which of the following disorders as a result of her current condition?

This patient likely has panic disorder, which presents with distressing cardiopulmonary/neurologic symptoms that reach their peak level of intensity in 10 minutes or less and then subside. Due to the intensity of the symptoms and their tendency to mimic a heart attack, patients may fear they are dying and seek treatment in the emergency department. Patients with panic disorder often develop agoraphobia, which is characterized by anxiety about and avoidance of multiple situations where they may feel trapped and helpless in the event of a panic attack (eg, crowds, enclosed spaces, public transportation). For example, this patient may start avoiding bus rides due to fears of recurrent panic attacks. In severe cases of agoraphobia, patients may restrict their activities to the point that they leave home only with a companion or they become completely housebound. Agoraphobia is a distinct condition and can be diagnosed with or without the presence of panic disorder.

A 4-year-old boy is brought to the office due to a progressively worsening cough for the past 2 days that is productive of yellow sputum. A year ago, he was found to have bilateral lower-lobe pulmonary infiltrates on chest x-ray and was diagnosed with pneumonia. The patient has since had 2 additional episodes of pneumonia, each requiring antibiotics for improvement of symptoms. His current chest x-ray again reveals bilateral lower lobe infiltrates. Further testing shows a high chloride content in his sweat. Which of the following abnormalities is most likely to be seen in this patient?

This patient most likely has cystic fibrosis (CF), an autosomal recessive disease characterized by recurrent sinopulmonary infections, pancreatic insufficiency, and malabsorption. It is caused by several different mutations that affect the CF transmembrane conductance regulator (CFTR) gene. The most common CFTR gene mutation (found in approximately 70% of cases) is a 3-base pair deletion of phenylalanine at amino acid position 508 (ΔF508). This mutation causes impaired post-translational processing (eg, improper folding and glycosylation) of CFTR, which is detected by the endoplasmic reticulum. As a result, the abnormal protein is targeted for proteasomal degradation, preventing it from reaching the cell surface. Certain drugs (eg, lumacaftor) can partially correct this folding defect, leading to expression of functional CFTR.

A 3-year-old boy is brought to the emergency department because of high fevers and malaise for the last 4 days. His parents say that he began limping yesterday and seems to refrain from using his right leg. He has no recent history of travel or exposure to a person with similar symptoms. His temperature is 39.4 C (103 F). Passive range of motion does not elicit pain and no joint effusion is seen. However, the patient refuses to bear weight with his right lower extremity. Scintigraphy is most likely to reveal increased focal radiotracer uptake in which of the following areas?

This patient most likely has hematogenous osteomyelitis, a disease that predominantly affects children, particularly boys. It usually affects the metaphysis of long bones, as this region contains slow-flowing, sinusoidal vasculature that is conducive to microbial passage. Adults are less likely to develop hematogenous osteomyelitis in the long bones due to changes associated with epiphysial closure. Hematogenous osteomyelitis begins with a seeding event that causes an acute cellulitis of the bone marrow. The resulting inflammation within the confined boney space leads to increased intramedullary pressure, which compromises blood flow and forces infectious exudate through vascular channels into the cortex and periosteal region. Disruption of the periosteal blood supply further contributes to bone ischemia, leading to necrosis. Without treatment, the infection can progress to chronic suppurative osteomyelitis, a condition in which necrotic bone (ie, sequestrum) serves as a reservoir for infection and becomes covered by a poorly constructed shell of new bone called an involucrum. One or more sinus tracts develop to drain the purulent material into the soft tissue or out to the skin surface. Proper treatment includes antibiotic therapy and debridement of necrotic bone. The vertebral body is the most common location for hematogenous osteomyelitis in adults. This is also the location of Pott disease, osteomyelitis of the vertebral body caused by Mycobacterium tuberculosis.

A 35-year-old woman is evaluated for a 3-month history of appetite loss, fatigue, weight loss, and abdominal pain. She has no chronic medical conditions and takes no medications. While undergoing upper gastrointestinal endoscopy, the patient develops cardiovascular collapse and dies, despite aggressive resuscitative measures. On autopsy, increased pigmentation is noted around the nipple areola and anal and vaginal mucous membranes. Which of the following additional findings are most likely to be seen on autopsy?

This patient with a three month history of weight loss, abdominal pain, and fatigue developed cardiovascular collapse during a medical procedure (endoscopy), which suggests chronic adrenal insufficiency complicated by acute adrenal crisis. Severe physiologic stress (eg, infection, medical procedures) normally induces increased glucocorticoid production; however, patients with adrenal insufficiency are unable to increase production. Adrenal crisis (eg, hypotension, shock) is rapidly fatal if unrecognized. Adrenal insufficiency can be caused by dysfunction of either the adrenal glands (primary) or pituitary (secondary). This patient's concomitant skin hyperpigmentation is highly suggestive of chronic primary adrenal insufficiency (PAI), as the resulting cortisol deficiency leads to increased ACTH secretion. ACTH is derived from proopiomelanocortin (POMC), the same prohormone from which melanocyte-stimulating hormone (MSH) is derived. Increased ACTH also leads to increased MSH secretion, which results in hyperpigmentation. Autoimmune adrenalitis is the most common cause (>90%) of chronic PAI and occurs due to autoantibody formation, resulting in atrophy of the bilateral adrenal glands. It can occur as an isolated disorder or in association with other autoimmune syndromes (eg, hypothyroidism, vitiligo). Bilateral adrenal hemorrhage (Waterhouse-Friderichsen syndrome [WFS]) can cause PAI and adrenal crisis but is typically seen acutely in the setting of bacterial sepsis (eg, Neisseria meningitidis). However, hyperpigmentation occurs in chronic adrenal insufficiency and therefore would not be expected with WFS.

What is the role of the CAAT box?

This patient with chronic microcytic anemia requiring blood transfusions and a predominance of hemoglobins F and A2 on hemoglobin electrophoresis likely has beta-thalassemia, a hereditary blood disorder characterized by reduced β-globin chain production. The base sequence indicated by the bold red arrow in this patient's β-globin gene represents the CAAT box, a highly conserved (consensus) sequence that functions as a promoter of transcription in the eukaryotic genome. It is typically located 70-80 bases upstream from the transcription start site. The Hogness (TATA) box is a second promoter region in the eukaryotic genome and is seen just to the right of the CAAT box in the above image. The TATA box is generally located 25 bases upstream from the transcription start site. Both the CAAT box and the TATA box promote initiation of transcription by acting as binding sites for general transcription factors and RNA polymerase II

A 22-year-old woman comes to the hospital due to a 5-day history of nausea, constipation, and severe, poorly localized abdominal pain. She also reports anxiety, difficulty concentrating, poor sleep quality, and tingling of the limbs. The patient has had several similar episodes in the past. She does not take any medications or use tobacco, alcohol, or illicit drugs. The patient has been restricting her diet to lose weight. On examination, the abdomen is soft, nontender, and nondistended. Bowel sounds are decreased. The patient receives an intravenous infusion of a heme preparation that leads to rapid resolution of her symptoms. The improvement in symptoms is most likely due to treatment-induced downregulation of which of the following enzymes?

This patient with neurologic symptoms (eg, tingling, difficulty concentrating) and recurrent episodes of nonspecific abdominal pain likely has acute intermittent porphyria (AIP), an autosomal dominant disease characterized by porphobilinogen (PBG) deaminase deficiency. Management of AIP attacks includes infusion of hemin, which downregulates hepatic aminolevulinate (ALA) synthase (rate-limiting enzyme in hepatic pathway of heme synthesis). Heme, a porphyrin, is synthesized in the liver for use in the cytochrome p450 enzyme system and in the bone marrow for hemoglobin use (these 2 synthesis pathways are regulated differently). A deficiency in any of the enzymes responsible for porphyrin synthesis can result in porphyria. Clinical manifestations result from the accumulation of porphyrin precursors in blood, tissues, and urine. AIP attacks, which are characterized by acute abdominal pain and neurologic symptoms, are due to accumulation of ALA and PBG, generally resulting from a combination of 2 factors: 1. PBG deaminase deficiency (inherited) 2. ALA synthase induction, typically precipitated by certain medications (eg, phenobarbital, griseofulvin, phenytoin), alcohol use, smoking, progesterone (eg, puberty), or a low-calorie diet (as seen with this patient) Glucose or hemin inhibit ALA synthase and are used in AIP management. PBG deaminase deficiency by itself is generally not sufficient for development of AIP attacks (many patients with PBG deaminase deficiency are asymptomatic), and PBG deaminase enzyme activity is not directly affected by hemin infusion (Choice E).

If you have psychosis symptoms for >1 day but less than 1 month what is it called?

This patient's 2-week history of hearing voices and paranoid delusions is consistent with brief psychotic disorder. Onset is typically sudden in response to an overwhelming stressor. Diagnosis requires ≥1 psychotic symptoms (ie, delusions, hallucinations, disorganized speech, grossly disorganized or catatonic behavior) lasting >1 day and <1 month with eventual return to premorbid functioning. The diagnosis requires ruling out psychosis secondary to a mood disorder and psychosis due to substances or a medical condition. Symptom duration is a key feature in differentiating brief psychotic disorder from other psychotic disorders. If this patient's symptoms were to continue for >1 month but <6 months, the diagnosis of schizophreniform disorder would be indicated (Choice G). If the symptoms were to continue for ≥6 months, the accurate diagnosis would be schizophrenia

What is buspirone used for and what are the benefits

This patient's chronic anxiety about multiple issues and associated symptoms of muscle tension, impaired concentration, and fatigue are consistent with a diagnosis of generalized anxiety disorder (GAD). Buspirone is a nonbenzodiazepine anxiolytic used in the treatment of GAD that has no risk of dependence in contrast to the substantial risk associated with benzodiazepines. Buspirone acts as a partial agonist of the 5HT1A receptor. In contrast to benzodiazepines, which act at GABA-A receptors, buspirone primarily affects serotonin and has no muscle relaxant or anticonvulsant properties. Buspirone is also not effective in treating panic disorder. Buspirone has fewer sedative and hypnotic effects compared to benzodiazepines and does not cause euphoria. Buspirone is not useful in the acute setting because it requires up to 2 weeks for the anxiolytic effect to begin.

A 28-year-old man is hospitalized due to the acute onset of severe depression and suicidal ideation for several days. He was recently thrown out of his mother's home after stealing her money to support his drug habit. The patient reports fatigue and vivid, disturbing dreams. The nursing staff note that he is irritable, withdrawn, hypersomnolent, and hyperphagic. The patient has an extensive history of substance abuse and has been hospitalized previously for alcohol detoxification. Blood pressure is 110/80 mm Hg and pulse is 64/min. Physical examination shows old injuries from a motorcycle accident but no other abnormalities. On mental status examination, he appears lethargic and dysphoric. This patient's current symptoms are most likely the result of which of the following conditions?

This patient's depression, fatigue, hypersomnia, hyperphagia, and vivid dreams are characteristic of cocaine withdrawal. Symptoms occur within hours to days of cessation or reduction of heavy or prolonged use. Following a binge, patients may experience a period of acute, intense symptoms, including severe depression with suicidal ideation, pronounced lassitude (the "crash"), and drug cravings. In contrast to withdrawal from opiates, alcohol, and benzodiazepines, withdrawal from cocaine and other stimulants usually results in minor physical symptoms. No medication has proven to be effective in treating cocaine withdrawal, and treatment is supportive only.

A 38-year-old man is brought to the emergency department by ambulance due to severe abdominal pain and undergoes an emergency appendectomy. Following a successful operation and recovery, the patient questions whether the appendectomy was absolutely necessary. He doubts the surgeon's explanation that the surgery was potentially lifesaving, angrily suggests that the hospital is taking advantage of his excellent medical insurance, and makes vague threats to sue the hospital. He asks to be discharged immediately, and explains, "I've earned everything I have on my own, and this hospital isn't getting any of it." When asked if he has any relatives or friends the team can speak with, he says, "I haven't talked to my family in 20 years, and I wouldn't call anyone my friend. Even if I did, that's none of your business." Which of the following is the most likely explanation of this patient's behavior?

This patient's distrust, suspicion about the surgeon's motives, and lack of sustained relationships are suggestive of paranoid personality disorder. Patients with this disorder exhibit a pattern of pervasive distrust of others beginning in early adulthood. These individuals tend to have minimal interpersonal relationships due to doubts about others' intentions and concerns that personal information will be used against them. Although paranoid personality disorder involves paranoid interpretations of benign comments and events, it can be differentiated from psychotic disorders by the lack of persistent, well-developed delusions and other psychotic symptoms (eg, hallucinations, disorganized speech or behavior).

What are the treatments for bipolar disease besides Lithium?

This patient's euphoric mood, decreased need for sleep, hyperactivity, grandiosity, and pressured speech lasting more than a week are characteristic of an acute manic episode of bipolar disorder. Bipolar disorder is a highly recurrent illness requiring maintenance treatment with mood-stabilizing medications to decrease the risk of recurrent mood episodes. Preferred medications for bipolar maintenance treatment include lithium, the anticonvulsants valproate and lamotrigine, and the second-generation antipsychotic quetiapine.

How many months are required for a ddx of Generalized anxiety disorder?

This patient's excessive chronic anxiety, muscle tension (tension headaches, neck pain), sleep disturbance, fatigue, and difficulty concentrating all support a diagnosis of generalized anxiety disorder (GAD). Patients with GAD experience excessive and uncontrollable worry about multiple issues. A symptom duration of ≥6 months is required for diagnosis, but many patients with GAD describe lifelong anxiety. In addition to muscle tension, other somatic symptoms are commonly seen (eg, sweating, gastrointestinal distress) and may prompt the patient to seek medical attention. First-line treatment includes cognitive-behavioral therapy and antidepressants (ie, selective serotonin reuptake inhibitors, serotonin-norepinephrine reuptake inhibitors).

In a patient with narcolepsy which of the following compounds is most likely to be abnormal in this patient's cerebrospinal fluid?

This patient's excessive daytime sleepiness, episodic loss of motor tone triggered by emotion (cataplexy), and inability to move on awakening (sleep paralysis) are highly suggestive of narcolepsy. Narcolepsy with cataplexy is most often caused by the lack of 2 related neuropeptides: hypocretin-1 (orexin-A) and hypocretin-2 (orexin-B), which are produced in neurons located in the lateral hypothalamus. These neuropeptides promote wakefulness and inhibit REM sleep-related phenomena. Hypocretin-1 levels are low to absent in the cerebrospinal fluid (CSF) of patients who have narcolepsy with cataplexy.

Differerntiate somatic disorder from illness anxiety disorder

This patient's excessive worry about having pancreatic cancer despite negative medical evaluation is consistent with illness anxiety disorder (IAD, [hypochondriasis]). Patients with IAD are preoccupied with fears of having a serious, undiagnosed illness and are rarely reassured by negative findings on physical examination or laboratory testing. Their fears of illness often become a dominant feature of their lives and lead to high health care utilization (eg, doctor shopping, requests for repeated testing). These patients often have catastrophic interpretations of normal physical sensations (eg, this patient thinking that stomach noises could signify cancer). IAD is differentiated from somatic symptom disorder in that patients with IAD have minimal or no somatic symptoms. In contrast, patients with somatic symptom disorder have prominent and typically multiple somatic symptoms. Initial management of IAD consists of scheduling regular visits, limiting diagnostic tests and referrals, and focusing on coping and functional improvement.

A 48-year-old man comes to the office due to concern that his skin is excessively dry, red, and cracked. The patient explains that he washes his hands each time he touches something due to fear of contamination. He spends 3-4 hours a day washing his hands and showers multiple times daily. He was recently fired from his job after refusing to touch keyboards shared by coworkers and is worried that he will be unable to find employment. On physical examination, the palms are erythematous with peeling skin. Treatment of this patient's disorder is most likely to involve a medication affecting which of the following neurotransmitters?

This patient's fear of contamination leading to compulsive washing rituals is consistent with obsessive-compulsive disorder (OCD). OCD is characterized by obsessions or compulsions, with most patients experiencing both. Common themes include contamination obsessions with cleaning compulsions; obsession with symmetry and compulsions involving ordering and counting; and fear of harm with checking compulsions (eg, stove off, doors locked). Patients with OCD typically engage in time-consuming rituals (>1 hr/day) that cause significant distress and/or functional impairment. Patients with washing compulsions may present with skin conditions. Selective serotonin reuptake inhibitor (SSRI) antidepressants are considered first-line treatment for OCD. SSRIs block the reuptake of serotonin into the presynaptic neuron, leading to an immediate increase in availability of synaptic serotonin and a subsequent cascade of downstream neurobiological effects.

What is the mutation in Achondroplasia and what is the inheritance pattern

This patient's limb shortening with normal torso length, specific craniofacial features (frontal bossing, midface hypoplasia), and risk of passing the condition to her offspring (50%) are characteristic of achondroplasia. The condition is caused by an autosomal dominant point mutation in the fibroblast growth factor receptor 3 (FGFR3) gene; 90% are de novo (unaffected parents) and 10% are inherited. Because the mutation is autosomal dominant, the fetus of an affected and an unaffected parent has a 50% chance of inheriting the mutation. Achondroplasia affects endochondral ossification, the process that makes long bones (eg, humerus, femur, phalanges) and portions of the skull and face. In endochondral bone formation, mesenchymal cells differentiate into chondrocytes that secrete cartilage matrix. These chondrocytes continue to proliferate through childhood, forming an elongating cartilage template that progressively becomes calcified and later invaded by osteoblasts, osteoclasts, and blood vessels originating from the periosteum. Deposition of osteoid matrix over the septa of calcified cartilage matrix forms woven bone, which then undergoes remodeling into compact bone. FGFR3 is normally responsible for limiting chondrocyte proliferation during endochondral ossification. In achondroplasia, FGFR3 becomes constitutively activated, causing exaggerated inhibition of chondrocyte proliferation that results in the characteristic bone shortening and craniofacial abnormalities.

A 35-year-old woman comes to the office for evaluation of anxiety. The patient works as a mechanical engineer and recently declined a promotion to be a project manager. Although her salary would have increased significantly, she felt that overseeing other employees and leading team meetings would be too stressful. She says, "If something were to go wrong, everyone would blame me. I don't even know why they offered me this position." The patient enjoys her job but tends to eat lunch by herself because she feels that her coworkers do not like her appearance or sense of humor. She has never had a long-term romantic partner but fantasizes about getting married someday. The patient lives alone, rarely socializes with friends, and is close with her mother, whom she describes as "the person I can always rely on no matter what." Which of the following is the most likely diagnosis?

This patient's pattern of avoiding social interactions and hypersensitivity to criticism is characteristic of avoidant personality disorder. Patients with this disorder typically have very limited social relationships due to the fear of being judged, embarrassed, or rejected. They desire social acceptance and may fantasize about having relationships, as in this patient; however, they struggle with feelings of inadequacy and pursue relationships only if certain of absolute acceptance by others. Occupational dysfunction due to difficulties interacting with coworkers or turning down promotions due to fear of criticism is common.

A 12-year-old girl is brought to the office for an annual checkup. The mother expresses concern about her daughter's behavior and poor grades, and says, "She is stubborn, irritable, overly emotional, and doesn't listen. She also seems resentful of her twin brother and deliberately annoys or upsets him. I can't get her to do chores or her homework, or go to bed on time." Over the past year, the mother has been called several times by the school guidance counselor about her daughter talking back to teachers and skipping classes. The patient says that school is "boring" and blames her behavior on "pointless rules and unreasonable demands," saying that her teachers are all "stupid." Which of the following is the most likely diagnosis in this patient?

This patient's pattern of deliberately annoying behavior and defiance of rules and authority figures that causes problems at home and at school suggests a diagnosis of oppositional defiant disorder (ODD). Blaming failures on others and being angry, argumentative or resentful are also characteristic of the disorder. Although the behavior often manifests with family members, it must also be observed during interactions with other individuals. For diagnosis, the oppositional behaviors must be excessive in frequency and duration when compared to normative age-appropriate behaviors. Although adolescents may test boundaries and assert their independence, this patient's 1-year history of persistent defiance, skipping classes, repeated refusal to follow rules, and irritable mood are beyond what would be expected for a 12-year-old girl

Two months ago, the patient was robbed and hit in the head and knees with a baseball bat when he resisted. His wife and other family members say that his overall demeanor has changed significantly since the attack. Prior to the assault, the patient was kind, considerate, and extremely polite. However, since the attack, he has been very irritable and rude, and was recently fired from his job for insulting customers and making socially insensitive comments to coworkers. Damage to which anatomical region of the brain is the most likely explanation for this patient's symptoms?

This patient's personality change, disinhibition, and irritability are most likely due to damage affecting the orbitofrontal cortex (OFC). The OFC is located in the frontal lobe and has strong modulatory connections to the limbic system (ie, the brain's primary emotional system); it is involved in behavioral and emotional regulation. Pathological behavioral and emotional changes are more commonly seen with bilateral, rather than unilateral, injury to the OFC. Disinhibition may be associated with significant impulsivity and loss of social etiquette, whereas increased irritability in the context of disinhibition may lead to frank, aggressive behavior. Diagnostic workup includes obtaining information from family and friends about personality changes, neuropsychological testing, and structural brain imaging.

A 35-year-old woman comes to the office due to insomnia and fatigue over the past month. She has felt increasingly depressed, irritable, and worthless since being let go from her job 5 weeks ago. Over the last month, the patient has lost 3.6 kg (8 lb) and has felt unmotivated and unable to concentrate on looking for new work. Other medical conditions include hypothyroidism and migraine headaches. The patient drinks 1 or 2 glasses of wine a few times a week and smokes marijuana once a month. Medications include levothyroxine and naproxen. Physical examination is normal. TSH level is 0.9 µU/mL. Which of the following is the most likely diagnosis in this patient

This patient's symptoms (eg, depressed mood, feelings of worthlessness, low energy, insomnia, weight loss, and poor concentration) lasting ≥2 weeks meets the criteria for major depressive disorder (MDD). The presence of MDD criterion symptoms for ≥2 weeks is all that is required for diagnosis, but the median duration of an MDD episode is approximately 6 months. Accurate diagnosis of MDD requires ruling out medical and substance-induced causes. Despite a history of hypothyroidism in this patient, her normal TSH level indicates that her hypothyroidism is adequately controlled with levothyroxine and would not be considered a medical cause of her depression Patients with sufficient depressive symptoms are diagnosed with MDD even if there is a clear psychosocial stressor that precipitated the depression (ie, this patient meets the criteria for MDD and therefore cannot be diagnosed with an adjustment disorder). In patients who do not meet the criteria for MDD, adjustment disorder with depressed mood may be diagnosed if sadness, distress, and functional impairment develop ≤3 months following a psychosocial stressor.

A 43-year-old man comes to the office with muscle weakness. His symptoms began 6 weeks ago and are worst in the hips and shoulders. The weakness has had a progressive course so that he is now having difficulty rising from chairs and combing his hair. The patient has had to reduce his regular exercise regimen due to weakness and has requested a modified work schedule for his job as a building maintenance manager. Muscle biopsy reveals major histocompatibility complex class I molecule overexpression on the sarcolemma with CD8+ lymphocyte infiltration. Which of the following is the most likely cause of this patient's condition?

This patient, a middle-aged man with progressive, symmetric proximal muscle weakness, has typical symptoms of polymyositis. The weakness may be painless or associated with diffuse myalgias. Muscle enzyme levels (eg, creatine kinase) are invariably elevated, and autoantibodies (eg, antinuclear antibodies, anti-Jo-1 antibodies) are present in most cases. Polymyositis is similar to dermatomyositis, although it lacks the typical skin findings; both may occur independently or as a paraneoplastic manifestation of an underlying malignancy. Polymyositis is an inflammatory myopathy triggered by unknown, possibly viral, antigens. It likely represents a cell-mediated immune response against myocytes. Increased expression of major histocompatibility complex class I antigens on the sarcolemma has been demonstrated and likely leads to presentation of autoantigens to CD8+ cytotoxic cells that subsequently initiate myocyte destruction. Muscle biopsy in polymyositis reveals inflammation, patchy necrosis, and regeneration and fibrosis of muscle fibers. Infiltration of the endomysium by macrophages and CD8+ lymphocytes is typically seen.

A 35-year-old man comes to the clinic with acute right knee pain and swelling. His symptoms have been present for a week and have moderately worsened over this period. The patient is able to bear weight but has significant pain when climbing stairs or walking for extended distances. He attempted treatment with ibuprofen, which provided prompt but only temporary relief. Past medical history is unremarkable, though the patient was seen by his primary care provider for a diarrheal illness 2 weeks before onset of the current symptoms. Examination shows a moderate-sized effusion at the right knee. Cultures of a joint aspirate reveal no bacteria. Which of the following is most likely associated with this patient's joint symptoms?

This patient, a young man with acute lower extremity arthritis and a sterile joint effusion following a gastrointestinal infection, has typical symptoms of reactive arthritis. Reactive arthritis is a spondyloarthropathy that preferentially affects HLA-B27-positive individuals. It typically presents as an asymmetric arthritis of the large joints. Common associated symptoms include conjunctivitis, urethritis, and keratoderma blennorrhagicum. Reactive arthritis is most common following infections with Campylobacter, Shigella, Salmonella, Yersinia, Chlamydia, or Bartonella. The clinical manifestations are caused by immune complexes involving bacterial antigens. However, it does not represent disseminated infection, and joint aspirates are sterile (ie, it is a 'reactive' not infectious arthritis).

Damage to tight junctions occurs from what microorg

Tight junctions are comprised of claudins and occludin and serve as paracellular barriers to water and solutes. The enterotoxin from Clostridium perfringens, a common cause of food poisoning, binds claudin and interferes with tight junctions in the intestinal barrier. Water loss from the tissue to the intestinal lumen results in watery diarrhea.

Pt undergoes intubation after premedication with muscle relaxant X and an appropriate sedative. The patient is then started on mechanical ventilation; however, he remains apneic longer than expected. Anesthesiology is consulted, and the patient's neuromuscular blockade is assessed by electrically stimulating a peripheral nerve 4 times in quick succession and observing the muscular response. The procedure is repeated 30 minutes later and the results show that initial response was symmetric rise, but 30 mins later it shows a progressive reduction of the responses. What muscle relaxant was used?

Train-of-four (TOF) stimulation is used during anesthesia to assess the degree of paralysis induced by neuromuscular junction (NMJ)-blocking agents. A peripheral nerve is stimulated 4 times in quick succession and the muscular response is recorded. The height of each bar represents the strength of each twitch; higher bars indicate the activation of increasing numbers of individual muscle fibers (myocytes). When a nondepolarizing NMJ blocker (eg, vecuronium) is administered, competitive inhibition of postsynaptic acetylcholine receptors at the motor endplate prevents some of these fibers from activating, decreasing the strength of the twitch. TOF stimulation shows a progressive reduction in each of the 4 responses (fading pattern) as a result of less acetylcholine being released with each subsequent impulse (due to the additional effect of presynaptic acetylcholine receptor blockade). In contrast, depolarizing blockers (eg, succinylcholine) initially function by preventing repolarization of the motor endplate and show equal reduction of all 4 twitches during TOF stimulation(phase I blockade). The responses remain equal because the presynaptic acetylcholine receptor stimulation helps to mobilize presynaptic acetylcholine vesicles for release. Persistent exposure to succinylcholine results in eventual transition to phase II blockade as the acetylcholine receptors become desensitized and inactivated (ie, functionally similar to nondepolarizing blockade). Succinylcholine is commonly administered for rapid-sequence intubation due to its rapid onset (<1 minute). The duration of action is determined by its metabolism by plasma cholinesterase and is typically <10 minutes. However, some patients are homozygous for an atypical plasma cholinesterase, which breaks down succinylcholine more slowly. In these patients, the paralysis can last for hours and they must be maintained on mechanical ventilation until spontaneous respirations resume.

A researcher is investigating the structure of the sarcomere using skeletal muscle obtained from an experimental animal. He develops monoclonal antibodies directed against a specific skeletal muscle protein and finds that these antibodies disrupt the binding of actin to structural support elements within the sarcomere. What part of the skeletal muscle sarcomere do these antibodies bind?

Z line A sarcomere is composed of overlapping filaments of actin and myosin as well as structural and binding proteins (eg, titin, α-actinin). A single sarcomere is defined as the distance between two Z lines. Thin (actin) filaments are bound to structural proteins at the Z line. The unbound ends of the actin filaments project into the middle of the sarcomere, where they interact with thick (myosin) filaments during muscle contraction.

A 26-year-old woman is being evaluated for a possible inherited disorder. She has a 6-year history of generalized tonic-clonic seizures, and a year ago, she had partial loss of vision due to an occipital infarction. Her mother has chronic intermittent muscle weakness and lactic acidosis, and her maternal uncle has hemiplegia. Skeletal muscle biopsy of the patient shows ragged-appearing muscle fibers. After further evaluation, all the affected family members are found to suffer from the same inherited disease. Pedigree analysis shows the dz pattern being transmitted from mother to all kids. What most likely explains the variability of clinical manifestations in the affected family members?

he neuromuscular lesions, ragged skeletal muscle fibers, and lactic acidosis in these family members suggest mitochondrial encephalomyopathy. Mitochondrial disorders follow a maternal inheritance pattern because an embryo's mitochondria are inherited from the ovum only. Mitochondria are responsible for ATP production via oxidative phosphorylation, which is why mitochondrial defects tend to cause lactic acidosis and primarily affect tissues with the highest metabolic rates (eg, neural tissue, muscular tissue). Although many mitochondrial proteins are coded for in the nuclear genome, mitochondria also contain their own genome, which is vulnerable to mutations. Each cell has hundreds of mitochondria, and defects in their genome may occur in any number of the mitochondria within the cell. Heteroplasmy describes the condition of having different mitochondrial genomes within a single cell. The severity of mitochondrial diseases is often directly related to the proportion of abnormal to normal mitochondria within a patient's cells.


Related study sets

CyberCollege TV Production Modules 46 - 48

View Set

Chapter 7: Measuring Output and National Income

View Set

Study Plan/Extra Questions - Metabolism, Osteoporosis, and Thyroid Disease

View Set

ME460 - pneumatics & hydraulic actuators

View Set

PSYC6 lecture 8: Conformity (1) & Pluralistic Ignorance (2)

View Set